Approach to Acutely Confused Patient (2025)

by Mehnaz Zafar Ali

You Have New Patients!

Patient 1

You meet a 40-year-old man in the ED, held by three security staff, looking diaphoretic and agitated, having tachycardia, and pointing vaguely in a direction as if interacting with imaginary people. When you try to assess him, he appears to be confused and disoriented and smells of alcohol. Over 6 hours, the patient has tremulousness, gets easily frightened, and gets further uncooperative for examination.

The image was produced by using ideogram 2.0.

Patient 2

You evaluate an 80-year-old woman in the ICU. She has a history of diabetes mellitus, hypertension, depression, and a stroke two years ago. She was admitted due to increased sleepiness, urinary and fecal incontinence for one week, and difficulty recognizing people. Before her admission, she was active and independent, had a reasonably good memory, and could manage household responsibilities. On physical examination, her eyes remain spontaneously closed but open with audible stimuli, and she is disoriented to time, place, and person.

The image was produced by using ideogram 2.0.

Introduction

Delirium is a rapidly developing clinical syndrome characterized by alterations in attention, consciousness, and awareness, with a reduced ability to focus, sustain, or shift attention. It commonly occurs in the elderly, with an incidence reported in 10% to 30% of patients hospitalized for medical illnesses and up to 50% following high-risk procedures [1].

This condition is also referred to as acute organic brain syndrome, characterized by rapid onset, diurnal fluctuations, and a duration of less than six months. Its behavioral presentation can vary, with the following manifestations.

  • Hyperactive Delirium: Patients present with increased agitation and heightened sympathetic activity. They may exhibit hallucinations, delusions, and combative or uncooperative behavior.

  • Hypoactive Delirium: Patients display increased somnolence and reduced arousal. The diagnosis is often overlooked due to its subtle clinical manifestations, which are frequently mistaken for fatigue or depression. This subtype is associated with higher rates of morbidity and mortality.

  • Mixed Presentation: Patients fluctuate between hyperactive and hypoactive delirium.

Delirium tremens (DT) is the most severe form of alcohol withdrawal syndrome and can be fatal. It typically occurs within 2 to 4 days following complete or significant abstinence from heavy alcohol consumption in approximately 5% of patients, with mortality rates as high as 50%. Alcohol functions as a depressant, similar to benzodiazepines and barbiturates, and affects serotonin and gamma-aminobutyric acid type A (GABA A) receptors, leading to tolerance and habituation.

Delirium is a dangerous and often preventable condition, associated with significant costs and increased morbidity and mortality. Among delirium patients presenting to the emergency department, there is a 70% increased risk of death within six months. In the ICU, delirium is linked to a 2- to 4-fold increased risk of overall mortality. Prevention, early diagnosis, and treatment of the underlying cause, along with well-coordinated care, are essential to improve patient outcomes.

General Approach

The diagnosis of delirium is primarily clinical and relies on careful history-taking, mental status examination, and detailed cognitive assessment. While laboratory and diagnostic tests may assist in identifying the underlying etiology, the initial evaluation should focus on addressing reversible causes. Life-threatening conditions must be promptly recognized, requiring rapid intervention and stabilization.

Differential Diagnoses

Delirium can present with symptoms that may be easily mistaken for mental illness, such as acute aggression, irritability, restlessness, and visual hallucinations [1]. Delirium mimics may include psychosis or mood disorders in the case of hyperactive delirium, and depression in the case of hypoactive delirium.

According to the International Classification of Diseases (ICD-10) guidelines [2], a definite diagnosis of delirium requires the presence of symptoms (mild or severe) in each of the five described areas. These include: impairment of consciousness and attention (ranging from clouding to coma, with a reduced ability to direct, focus, sustain, and shift attention), global disturbance of cognition, psychomotor disturbances, disturbance of the sleep-wake cycle, and emotional disturbances.

Delirium

Delirium typically presents with an acute onset and progresses rapidly. It often resolves completely with treatment of the underlying cause. Clinically, it is characterized by fluctuating levels of consciousness, inattention, disorientation, worsening symptoms in the evening (a phenomenon known as sundowning), and transient visual hallucinations. Delirium carries significant risks, including high mortality due to the underlying medical condition, as well as increased risk of falls, injuries, exhaustion, or aggression.

Dementia

Dementia has an insidious onset and follows a chronic, progressive course marked by continuous deterioration over time. Key clinical features include memory disturbances, changes in personality or behavior, apathy, and apraxia. Individuals with dementia are at risk of falls, neglect, abuse, agitation, and wandering away from their safe environments.

Depression

Depression typically has a slow onset and an episodic course, with periods of remission and recurrence. Symptoms include a persistently depressed mood, loss of interest or pleasure in activities, reduced energy, feelings of hopelessness, disturbances in sleep and appetite, difficulties with concentration, and pervasive negative thoughts, often accompanied by guilt. The associated risks include suicide, deliberate self-harm, neglect, and agitation.

Psychosis

Psychosis usually begins insidiously and follows a progressive course punctuated by episodes of exacerbation. Clinical features include delusions, auditory hallucinations, disorganized thoughts, social withdrawal, apathy, avolition (lack of motivation), and impaired reality testing. Psychosis poses risks such as aggression, harm to others, and non-adherence to treatment, which can exacerbate the condition further.

History and Physical Examination Hints

It is of paramount importance to obtain a detailed corroborative history regarding the onset, course, and progression of the illness, along with performing a thorough physical and neurological examination of the patient. A biopsychosocial formulation must identify the predisposing, precipitating, and perpetuating causes of delirium [1].

The mnemonic “I WATCH DEATH,” developed by Dr. M.G. Wise in 1986, is a valuable tool for clinicians to screen for possible causes of delirium [3].

  • I – Infections: Infections are a common cause and can include conditions such as sepsis, urinary tract infections, encephalitis, and meningitis.

  • W – Withdrawal: Sudden withdrawal from substances such as alcohol, sedatives, or drugs can lead to significant medical complications.

  • A – Acute Metabolic Disturbances: Issues such as electrolyte imbalances (e.g., hyponatremia) and organ failure, such as hepatic or renal failure, can significantly disrupt normal physiological functions.

  • T – Trauma: Physical injuries, including head trauma and falls, are notable causes that may lead to further complications like bleeding or swelling.

  • C – CNS Pathology: Central nervous system disorders such as stroke, hemorrhage, seizures, or the presence of space-occupying lesions like tumors can have profound impacts on a patient’s condition.

  • H – Hypoxia: A lack of adequate oxygen supply, often due to anemia or hypotension, can result in significant systemic effects.

  • D – Deficiencies: Nutritional deficiencies, particularly a lack of essential vitamins and minerals like thiamine, can result in various clinical symptoms.

  • E – Endocrine Disorders: Hormonal imbalances, including thyroid storm and hyperglycemia, can disrupt metabolic processes and cause severe systemic effects.

  • A – Acute Vascular Events: Sudden vascular events, such as subarachnoid hemorrhage, require prompt identification and management due to their life-threatening nature.

  • T – Toxins or Drugs: Exposure to industrial poisons, carbon monoxide, or drugs with anticholinergic properties can have toxic effects on the body.

  • H – Heavy Metal Poisoning: Exposure to heavy metals such as lead and mercury can lead to chronic toxicity and require specific interventions.

Several factors increase the likelihood of developing delirium, especially in vulnerable populations:

  1. Age: Both elderly individuals and young children are at heightened risk due to their increased susceptibility to physiological and cognitive changes.

  2. Recent Hospitalizations: Hospital stays, particularly those involving medical illnesses or surgical procedures, can act as significant stressors and predispose individuals to delirium.

  3. Pre-existing Brain Conditions: Conditions like brain damage or dementia further increase the risk, as they impair cognitive resilience.

  4. Chronic Medical Disorders: Long-term health conditions often contribute to a state of chronic physiological stress, increasing the likelihood of delirium.

  5. Sensory Deprivation: Impairments in vision or hearing can lead to sensory deprivation, which may exacerbate confusion and disorientation.

  6. Substance Use Disorders: Alcohol or drug use disorders are major contributors to the onset of delirium, particularly during withdrawal periods or intoxication.

  7. Medications: The use of psychotropic medicines and polypharmacy (simultaneous use of multiple medications) heightens the risk of delirium due to potential drug interactions and side effects.

  8. History of Delirium: Individuals with a previous history of delirium are more likely to experience recurrent episodes, particularly if the underlying risk factors persist.

  9. Malnutrition: Poor nutritional status can exacerbate vulnerability to delirium by impairing metabolic and neurological functions.

  10. Burns: Severe burns create systemic inflammation and stress, which can predispose individuals to delirium.

Screening tools for delirium, such as the Mini-Mental Status Examination (MMSE) [4] and the Confusion Assessment Method (CAM) [5], are valuable for early identification and intervention. These tools can also be used to monitor clinical improvement when performed repeatedly during the course of the illness.

The Confusion Assessment Method (CAM) includes four key features to identify delirium. A diagnosis of delirium requires the presence of Features 1 and 2 and either Feature 3 or Feature 4:

Feature 1 – Acute Onset and Fluctuating Course: There is evidence of an acute change in mental status from the patient’s baseline.
The abnormal behavior fluctuates throughout the day, tending to come and go or change in severity.

Feature 2 – Inattention: The patient has difficulty focusing attention, is easily distractible, or cannot keep track of what is being said.

Feature 3 – Disorganized Thinking: The patient demonstrates disorganized or incoherent thinking, such as rambling or irrelevant conversation, illogical flow of ideas, or unpredictable switching between subjects.

Feature 4 – Altered Level of Consciousness: The patient’s consciousness level deviates from “alert.” It may range from hyperalert (vigilant) to lethargy, stupor, or coma.

The CAM is a widely used, reliable tool with high sensitivity (94–100%) and specificity (90–95%). It enables quick and accurate identification of delirium, facilitating early intervention to manage underlying causes and improve patient outcomes.

Confusion Assessment Method (CAM) Instrument:

  1. Acute Onset:
    • This involves an abrupt change in the patient’s mental status, which is evident when comparing their current state to their baseline cognitive function. This change may be noticed by family members, caregivers, or clinicians and is typically indicative of an acute underlying medical issue or condition.
  2. Inattention:
    • 2A: The patient has difficulty concentrating or paying attention. This may manifest as being easily distracted, unable to follow conversations, or losing track of what is being discussed.
    • 2B: If inattention is present, the behavior often fluctuates over time, meaning it can improve or worsen during an assessment or throughout the day.
  3. Disorganized Thinking:
    • The patient’s thought process appears chaotic or incoherent. They may exhibit rambling, irrelevant speech, an illogical sequence of ideas, or rapid, unpredictable topic changes during a conversation. This suggests a loss of organized, goal-directed thinking.
  4. Altered Level of Consciousness:
    • The patient’s alertness deviates from normal. This can range from:
      • Alert (normal): Fully awake and responsive.
      • Vigilant (hyperalert): Overly sensitive to stimuli, easily startled, or hypervigilant.
      • Lethargic: Drowsy but easily aroused.
      • Stupor: Difficult to arouse, with limited responsiveness to stimuli.
      • Coma: Unarousable and non-responsive.
  5. Disorientation:
    • The patient is confused about time, place, or identity. They may incorrectly believe they are in a different location, misjudge the time of day, or demonstrate an inability to recognize familiar surroundings or people.
  6. Memory Impairment:
    • Memory issues are evident when the patient cannot recall recent events, forgets instructions, or struggles to remember details of their hospital stay or interactions.
  7. Perceptual Disturbances:
    • The patient may experience hallucinations (e.g., seeing or hearing things that aren’t present), illusions (misinterpreting real stimuli, such as mistaking a shadow for an object), or misinterpretations (believing something benign, such as a coat rack, is threatening).
  8. Psychomotor Disturbances:
    • 8A (Agitation): The patient may exhibit increased motor activity, such as restlessness, repeatedly picking at bedclothes, tapping their fingers, or making frequent, sudden movements.
    • 8B (Retardation): Alternatively, the patient may show decreased motor activity, appearing sluggish, staring into space, staying in the same position for extended periods, or moving very slowly.
  9. Altered Sleep-Wake Cycle:
    • Disturbances in the patient’s sleep pattern are evident. They may experience excessive daytime sleepiness coupled with difficulty sleeping at night, or their sleep-wake rhythm may become reversed.

Associated Features

Certain medical conditions can present with a range of distressing symptoms and features:

  1. Hallucinations and Illusions: Patients may experience vivid and often frightening visual or auditory hallucinations. Additionally, tactile hallucinations, such as the sensation of insects crawling on the body, can occur, adding to their distress.

  2. Autonomic Disturbances: Marked autonomic instability is common and may include symptoms such as tachycardia, fever, hypertension, sweating, and pupillary dilation.

  3. Psychomotor and Coordination Issues: Psychomotor agitation and ataxia (lack of muscle coordination) are frequently observed, contributing to physical instability and difficulty performing tasks.

  4. Sleep Disturbances: Insomnia is a notable feature, often accompanied by a reversal of the sleep-wake cycle, further exacerbating cognitive and physical impairments.

It is crucial to obtain a detailed history of the patient’s premorbid personality, as this helps establish their baseline cognitive state and allows the clinician to determine the magnitude of cognitive deterioration. Patients with fluctuating levels of consciousness may experience rapid shifts in their activity levels, ranging from extreme psychomotor excitement to sleepiness during an interview [1].

The Mental State Examination (MSE) should include an assessment of mood (e.g., apathy, blunted affect, emotional lability), behavior (e.g., withdrawn, agitated), activity levels, thoughts (e.g., delusions), and perceptions (e.g., hallucinations, illusions). A brief cognitive assessment may utilize the COMA framework, which evaluates Concentration, Orientation, Memory, and Attention.

Clinical Institute Withdrawal Assessment of Alcohol Scale, Revised (CIWA-Ar)

The CIWA-R is a tool designed to standardize the assessment of withdrawal severity in patients experiencing alcohol withdrawal. This instrument is particularly useful for guiding treatment decisions and ensuring appropriate management of symptoms.

Alcohol withdrawal delirium progresses through distinct stages, including:

  • Tremulousness or Jitteriness: Occurs within 6–8 hours of cessation or reduction in alcohol use.
  • Psychosis and Perceptual Symptoms: Develops between 8–12 hours, marked by hallucinations and disorganized thinking.
  • Seizures: Typically occur within 12–24 hours of withdrawal.
  • Delirium Tremens: The most severe stage, manifesting within 24–72 hours and potentially lasting up to one week. This phase is characterized by confusion, autonomic instability, and significant risk of complications.

The CIWA-R plays a critical role in monitoring these stages and ensuring timely interventions to mitigate risks associated with alcohol withdrawal.

Click here to download full CIWA-R evaluation form.

Diagnostic Tests and Interpretation

Relevant laboratory tests and diagnostic imaging are recommended to assess the underlying etiology of delirium. Routine workups for electrolytes, kidney and liver function, and pregnancy tests for women are advised. Blood tests can help identify medical conditions that may mimic delirium, such as hypoglycemia and diabetic ketoacidosis (via blood sugar levels) or thyrotoxicosis (via thyroid profile). Test results indicative of long-term heavy alcohol use, such as evidence of cirrhosis or liver failure on ultrasound, macrocytic anemia, and elevated liver transaminase levels—particularly gamma-glutamyl transpeptidase—can aid in reaching the correct diagnosis [6].

Positron emission tomographic (PET) studies have suggested a globally low rate of metabolic activity, particularly in the left parietal and right frontal areas, in otherwise healthy individuals withdrawing from alcohol. Diffuse slowing of the background rhythm has been observed on electroencephalography (EEG) in patients suffering from acute delirium, except in cases of alcohol-related delirium tremens, which typically exhibit fast activity [1].

Management

Delirium is a medical emergency requiring immediate hospitalization to correct the underlying causes while minimizing risks associated with behavioral symptoms, aggression, dehydration, falls, and injury. High-potency antipsychotics in low doses are recommended for managing aggression and behavioral symptoms. Haloperidol (Haldol) has been extensively studied for reducing agitation due to delirium [7]. Evidence also supports the use of other atypical antipsychotics such as risperidone. Aripiprazole has demonstrated significant benefit in the complete resolution of hypoactive delirium [8].

The use of benzodiazepines should be restricted to cases of delirium caused by alcohol withdrawal. If liver function is not impaired, a long-acting benzodiazepine, such as chlordiazepoxide or diazepam, is preferred and can be administered orally or intravenously. In cases of reduced liver function, lorazepam may be given orally or parenterally as needed to stabilize vital signs and sedate the patient. These medications should then be tapered gradually over several days with close monitoring of vital signs. Anticonvulsants like carbamazepine and valproic acid are also effective in managing alcohol withdrawal. However, antipsychotics should be avoided in such cases due to their potential to lower the seizure threshold. Chronic alcoholics are at high risk of vitamin B1 (thiamine) deficiency, which can predispose them to Wernicke-Korsakoff syndrome (characterized by memory problems, confabulation, and apathy), cerebellar degeneration, and cardiovascular dysfunction. To mitigate this risk, such patients should receive 100 mg of thiamine intravenously before glucose administration.

Environmental modification strategies are particularly useful for managing delirious patients. These include providing well-illuminated rooms with good ventilation and reorientation cues such as calendars and alarm clocks. Assigning patients to a room near the nursing station allows for closer monitoring, ideally with the presence of a family member or close friend. In severe cases with agitation or injury risk, one-on-one supervision is advisable to ensure patient safety [1]. Both under-stimulation and overstimulation should be avoided. The use of physical restraints should be considered a last resort, with frequent monitoring and discontinuation as soon as possible. Psychoeducation for family members and caregivers is crucial to manage expectations and improve their involvement in the patient’s care [2].

Special Patient Groups and Other Considerations

Elderly patients are at high risk of altered mental status, and studies have recommended advanced age as an independent risk factor warranting screening of this vulnerable group through structured mental state assessments. It is important to recognize that behavioral manifestations of this magnitude should not be regarded as a normal part of the aging process. Dementia must be carefully differentiated from delirium in the geriatric population, as dementia typically presents with an insidious onset and a progressive course [3].

Other risk factors in the elderly that require attention include underlying neurological causes, multiple medical comorbidities, polypharmacy, poor drug metabolism, and sensory limitations [9]. Medications for elderly patients should be initiated at lower doses, and potential drug interactions must be considered whenever new medications are introduced.

The pediatric age group may present with nonspecific symptoms of acute onset, necessitating a detailed history and physical examination to rule out causes such as fever, injury, or foreign objects. Pregnancy, meanwhile, may predispose healthy women to medical conditions such as diabetes, venous thromboembolism, strokes, and eclampsia [9].

When To Admit This Patient

Admission decisions for confused patients or those undergoing alcohol withdrawal require a multifaceted approach that prioritizes accurate diagnosis, evidence-based treatment, and legal considerations. These decisions should aim to address the immediate medical needs while planning for long-term recovery and safety.

Admitting a confused patient requires careful evaluation of the underlying causes, as confusion can result from various conditions such as dementia, delirium, or depression, each requiring distinct management strategies [10]. Delirium, an acute confusional state, is particularly prevalent in older adults and often develops rapidly with fluctuating severity [11]. It is essential to determine whether the confusion is acute, chronic, or a combination of both, as this distinction guides the initial management plan [11].

Risk factors for acute confusion include admission from non-home settings, lower cognitive scores, restricted activity levels, infections, and abnormal laboratory values. These indicators suggest frailty and may also point to underlying chronic undernutrition or dehydration [12]. Early recognition and appropriate management are crucial to reducing morbidity and mortality, as confusion is often misdiagnosed or undertreated in hospital settings [10].

Furthermore, legal and ethical challenges, such as evaluating a patient’s decision-making capacity and ensuring that any necessary restraints are lawful and ethical, must be addressed to avoid infringing on the patient’s rights [13]. A comprehensive assessment of cognitive and physical status, coupled with an understanding of legal considerations, is essential for developing a management plan that effectively addresses the specific causes and risks associated with confusion [11-13].

Disposition decisions for confused patients, including those undergoing alcohol withdrawal, require a comprehensive and systematic approach that integrates accurate diagnosis, appropriate treatment, and continuous monitoring. Alcohol withdrawal can result in severe complications, such as seizures and delirium tremens, with mortality rates ranging from 1% to 30%, depending on the quality of treatment provided [14]. Prompt identification and management are critical, often involving benzodiazepines like diazepam to alleviate symptoms and prevent progression to life-threatening conditions [15]. Management becomes particularly challenging in critically ill patients, as incomplete alcohol consumption histories and the need for adjunctive medications beyond benzodiazepines complicate care during severe withdrawal or delirium tremens [16].

Emergency departments frequently encounter substance use disorders; however, less than half of alcohol-related issues are identified, highlighting the importance of comprehensive assessments and evidence-based interventions. Effective disposition decisions rely on early identification, tailored treatment strategies, and ongoing evaluations to ensure patient safety and recovery.

Clinical Pearls

  • Alcohol Withdrawal Characteristics: Alcohol withdrawal can begin within hours to days following heavy and prolonged alcohol use. A key feature of alcohol withdrawal is autonomic hyperactivity, which may present as increased heart rate, sweating, tremors, and other signs of sympathetic nervous system overactivity.
  • Overlap with Sedative-Hypnotic Withdrawal: The diagnostic criteria and symptoms for alcohol withdrawal are identical to those for sedative-hypnotic withdrawal. This similarity highlights the importance of carefully assessing a patient’s history of substance use to guide appropriate management.
  • Treatment Approaches:
    • Delirium Due to General Medical Conditions: The preferred treatment is low doses of high-potency antipsychotics, which help manage symptoms without excessive sedation or complications.
    • Alcohol Withdrawal: Benzodiazepines remain the first-line treatment to alleviate withdrawal symptoms and prevent complications such as seizures or delirium tremens. In cases where hepatotoxicity is a concern, short-acting benzodiazepines like lorazepam are preferred due to their safer profile in patients with compromised liver function.
  • Hallucinations and Diagnosis: Visual hallucinations are more characteristic of delirium than of primary psychiatric disorders. This distinction is critical in differentiating between medical and psychiatric causes of altered mental status.

Revisiting Your Patient

Patient 1

The image was produced by using ideogram 2.0.

The patient presents with the smell of alcohol and clinical features consistent with delirium tremens, a severe manifestation of alcohol withdrawal.

Further Management: The patient should be treated promptly with a benzodiazepine, starting with high doses and tapering as recovery progresses. Chronic alcohol users are commonly deficient in vitamin B1 (thiamine), which can result in dementia and cognitive impairments. Thiamine replacement should be administered prior to glucose to prevent the development of Wernicke-Korsakoff syndrome [17].

Patient 2

The image was produced by using ideogram 2.0.

The patient is unresponsive to stimuli, disoriented, and has multiple medical conditions, which is suggestive of delirium due to a general medical condition, hypoactive type.

Further Management: Immediate steps should include ensuring 24-hour supervision, investigating the underlying cause, and implementing reorientation strategies. Low-dose antipsychotics have been recommended, with studies reporting complete resolution of symptoms with the use of aripiprazole and other atypical antipsychotics [18].

Author

Picture of Mehnaz Zafar Ali

Mehnaz Zafar Ali

Consultant Psychiatrist, Al Amal Psychiatry Hospital, Emirates Health Services, Dubai, United Arab Emirates

Listen to the chapter

References

  1. Gleason OC. Delirium. Am Fam Physician. 2003;67(5):1027-1034.
  2. World Health Organization. Organic, including symptomatic, mental disorders. In: International Statistical Classification of Diseases and Related Health Problems. 10th ed. 2016:182-188.
  3. Gower LE, Gatewood MO, Kang CS. Emergency department management of delirium in the elderly. West J Emerg Med. 2012;13(2):194-201. doi:10.5811/westjem.2011.10.6654.
  4. Folstein MF, Folstein SE, McHugh PR.Mini-mental state”. A practical method for grading the cognitive state of patients for the clinician. J Psychiatr Res. 1975;12(3):189-198. doi:10.1016/0022-3956(75)90026-6.
  5. Inouye SK, van Dyck CH, Alessi CA, Balkin S, Siegal AP, Horwitz RI. Clarifying confusion: the confusion assessment method. A new method for detection of delirium. Ann Intern Med. 1990;113(12):941-948. doi:10.7326/0003-4819-113-12-941.
  6. Chan M, Moukaddam N, Tucci V. Stabilization and management of the acutely agitated or psychotic patient. In: Cevik AA, Quek LS, Noureldin A, Cakal ED, eds. International Emergency Medicine Education Project. 1st ed. iEM Education Project; 2018:452-457.
  7. Smit L, Slooter AJ, Devlin JW, et al. Efficacy of haloperidol to decrease the burden of delirium in adult critically ill patients: the EuRIDICE randomized clinical trial. Crit Care. 2023;27(1):413. doi:10.1186/s13054-023-04692-3.
  8. Lodewijckx E, Debain A, Lieten S, et al. Pharmacologic treatment for hypoactive delirium in adult patients: a brief report of the literature. J Am Med Dir Assoc. 2021;22(6):1313-1316.e2. doi:10.1016/j.jamda.2020.12.037.
  9. Cetin M, Oktem B, Canakci ME. Altered mental status. In: Cevik AA, Quek LS, Noureldin A, Cakal ED, eds. International Emergency Medicine Education Project. 1st ed. iEM Education Project; 2018:111-121.
  10. Winstanley L, Glew S, Harwood RH. A foundation doctor’s guide to clerking the confused older patient. Br J Hosp Med (Lond). 2010;71(5):M78-M81. doi:10.12968/hmed.2010.71.Sup5.47934.
  11. Andrews H, Clarke A, Parmar S, et al. You’ve been bleeped: the confused patient. BMJ. 2015;351:h3266. doi:10.1136/sbmj.h3266.
  12. Wakefield BJ. Risk for acute confusion on hospital admission. Clin Nurs Res. 2002;11(2):153-172. doi:10.1177/105477380201100205.
  13. Lyons D. The confused patient in the acute hospital: legal and ethical challenges for clinicians in Scotland. J R Coll Physicians Edinb. 2013;43(1):61-67. doi:10.4997/jrcpe.2013.114.
  14. Thanyanuwat R. Patients who suffer from alcohol withdrawal and disorientation. J Med Assoc Thai. 2013;96(2):78-83.
  15. Thompson WL. Management of alcohol withdrawal syndromes. Arch Intern Med. 1978;138(2):278-283. doi:10.1001/archinte.1978.03630260068019.
  16. Sutton LJ, Jutel A. Alcohol withdrawal syndrome in critically ill patients: identification, assessment, and management. Crit Care Nurse. 2016;36(1):28-40. doi:10.4037/ccn2016420.
  17. Toy EC, Klamen DL. Alcohol withdrawal. In: Case Files: Psychiatry. 6th ed. McGraw-Hill Education; 2020:400-405.
  18. Lodewijckx E, Debain A, Lieten S, et al. Pharmacologic treatment for hypoactive delirium in adult patients: a brief report of the literature. J Am Med Dir Assoc. 2021;22(6):1313-1316.e2. doi:10.1016/j.jamda.2020.12.037.

Reviewed and Edited By

Picture of Arif Alper Cevik, MD, FEMAT, FIFEM

Arif Alper Cevik, MD, FEMAT, FIFEM

Prof Cevik is an Emergency Medicine academician at United Arab Emirates University, interested in international emergency medicine, emergency medicine education, medical education, point of care ultrasound and trauma. He is the founder and director of the International Emergency Medicine Education Project – iem-student.org, chair of the International Federation for Emergency Medicine (IFEM) core curriculum and education committee and board member of the Asian Society for Emergency Medicine and Emirati Board of Emergency Medicine.

Carbon Monoxide Poisoning (2024)

by Mohammad Issa Naser & Abdulla Alhmoudi

You have a new patient!

A 48-year-old male with a known medical history of hypertension, depression, and prior suicidal attempts was brought into the Emergency Department by EMS after he was found unconscious by his wife after she arrived from work. He was lying down in an enclosed garage at home with the car engine running. She states that her husband was having difficulty breathing when she found him and was not responding to her. She reported that he had been depressed for the last few weeks because of financial problems. Upon arrival at the ED, the patient was unresponsive, with the following vital signs noted: BP 113/74 mmHg, HR 114 bpm, RR 10 bpm, and oxygen saturation at 98%.

a-photo-depicts-a-48-year-old-man-found-in-the-garage (image was created by using ideogram 2.0)

What do you need to know?

Importance and Epidemiology

Carbon Monoxide (CO) is often called the “silent killer” as it lacks any warning or alarming signs of its presence. It is a colorless, odorless, tasteless, and non-irritating gas formed by the incomplete combustion of hydrocarbon fuels.

Despite a historical decline in the number of cases, CO continues to be one of the major causes of poisoning-related ED visits, accounting for approximately 50,000 cases every year in the United States, with a mortality rate of 1% to 3% [1]. Although many of these are nonfatal exposures with various degrees of toxicity, an estimated 1,000 to 2,000 patients a year die from severe toxicity [2]. Intentional poisoning cases have higher mortality rates compared to accidental cases and account for two-thirds of deaths [3,4]. Although cases can occur around the year, CO poisoning has a seasonal and geographic relation with cold climates, peaking during winter months, most commonly from faulty furnaces [5].

CO poisoning often has nonspecific toxicologic presentations ranging from minimal symptoms to unresponsiveness. It requires higher suspicion from clinicians to recognize, diagnose, and provide timely and appropriate management to avoid morbidity, mortality, and long-lasting complications. ED physicians should always consider CO poisoning when multiple patients present to the ED from a single location with similar correlating findings [3].

Pathophysiology

CO poisoning causes tissue hypoxia by impairing oxygen delivery and utilization and generating reactive oxygen species. CO can rapidly diffuse into the pulmonary circulation and reversibly bind the iron moiety of heme with approximately 240 times the affinity of oxygen-forming carboxyhemoglobin (COHb). CO impairs heme’s ability to deliver oxygen by directly occupying oxygen-binding sites and causing a conformational change to the other three oxygen-binding sites. This allosteric change increases the affinity of the oxygen binding site and decreases the oxygen delivery to the peripheral tissues, causing a leftward shift in the oxyhemoglobin dissociation curve. The amount of carboxyhemoglobin formed depends on the amount of CO and oxygen in the environment, duration of exposure, and minute ventilation [6].

CO also binds to myoglobin and NADPH reductase, which can worsen the hypoxia of cardiac muscle by affecting the mitochondria and ATP production, potentially leading to atraumatic rhabdomyolysis [2]. Like cyanide, CO inactivates cytochrome oxidase, which is involved in mitochondrial oxidative phosphorylation, causing a switch to anaerobic metabolism, and their combined effects can be synergistic in smoke inhalation [7]. Other effects of CO poisoning include neutrophil degranulation, free radical formation, lipid peroxidation in the brain and other tissues, and cellular apoptosis [2,8]. The half-life of COHb is about 300 minutes; thus, it begins to accumulate in the blood within a short exposure time. With normobaric oxygen (NBO) therapy (which is 100% inhaled oxygen at normal atmospheric pressure), the half-life is decreased to between 50 and 100 minutes; with Hyperbaric oxygen therapy, the half-life can be reduced to 30 minutes [9,10].

Medical History

A thorough history can be very helpful for early recognition of CO-related poisoning. Clinical findings can be variable and highly unspecific. The most common complaint in patients with mild to moderate CO poisoning is headache, present in up to 58% of patients, followed by the wide range of unspecific findings of nausea, dizziness, drowsiness, vomiting, cough or choking, confusion, shortness of breath, syncope, throat and eye irritation and chest pain [3]. It is important for clinicians to inquire about potential CO sources such as residential heating systems, gas appliances, or recent fires. In addition, clinicians should specifically inquire about transient loss of consciousness, as the presence or absence of this finding can be important in determining the severity of the presentation and the need for further interventions like hyperbaric oxygen [6]. Delayed neurological sequelae (DNS) is a well-known complication and can occur in 15 to 40 percent of patients presenting with significant CO poisoning [11]. DNS has been reported to appear 3 to 240 days after apparent recovery, with the majority of cases occurring within 20 days of CO poisoning. Deficits can last a year or more and are typically not found on acute presentation. Patients may present with cognitive impairment, memory deficits, movement disorders, or psychiatric symptoms. Any neurological or neuropsychiatric symptoms persisting beyond the acute phase of CO poisoning should raise suspicion for DNS and warrant appropriate evaluation and management [6]. Risk factors that predict the development of delayed neurologic sequelae include extremes of age and loss of consciousness. Because most CO-poisoned patients reaching the ED survive with minimal intervention, prevention of delayed neurologic and neuropsychiatric sequelae is a primary goal of therapy [12].

Physical Examination

Physical examination in suspected CO poisoning patients should focus on vital signs, cardiac and pulmonary examination, and a thorough neurological assessment. Findings in CO poisoning are usually limited to changes in mental status, tachycardia, and tachypnea in the absence of history of trauma or burns. Symptoms can range from mild confusion to coma [6]. The presence of “cherry-red” skin or mucous membranes may be observed in severe cases or even noted postmortem. However, it’s neither a sensitive nor specific sign, and it does not exclude CO poisoning [13]. Severe CO poisoning can be associated with neurologic, metabolic, and cardiovascular red flags such as seizures, syncope, lactic acidosis, acute myocardial infarction, ventricular arrhythmia, and pulmonary edema [6].

Alternative Diagnoses

Carbon monoxide poisoning can be a “great mimic,” but the early presentations are often nonspecific and readily confused with other conditions, typically a viral syndrome, explaining why influenza is the most common misdiagnosis [14]. CO poisoning can also be misdiagnosed frequently as gastroenteritis, food poisoning, or even colic in infants. Like adults, children tend to develop nonspecific symptoms that complicate the diagnosis [15]. More severe poisoning may be confused with other causes of altered mental status, such as trauma, diabetic ketoacidosis, meningitis, hypoglycemia, and intoxication [16]. The differential diagnosis remains broad without a known exposure source or sick contacts as clues. Cyanide poisoning, especially in patients with smoke inhalation, should also be considered due to the potential for concurrent exposure. In cases of chronic CO exposure, chronic fatigue, mood disorders, sleep disorders, and memory problems should be considered as an alternate diagnosis [17]. Recognizing risk factors for CO poisoning can be crucial in determining the likelihood of CO poisoning; focusing on potential sources of CO poisoning, the presence of multiple individuals with similar symptoms from the same location increases the likelihood of CO poisoning. The CNS is the organ system most sensitive to CO poisoning. Acutely, otherwise healthy patients may manifest headache, dizziness, and ataxia at COHb level as low as 15% to 20%; with higher levels and longer exposures, syncope, seizures, or coma may result [15]. At the same time, history of consuming contaminated food or recent sick contact with flu-like symptoms would make the diagnosis less likely.

Acing Diagnostic Testing

The single most useful diagnostic test to use in a suspected CO poisoning is COHb levels.15 An arterial or venous blood gas analysis with elevated carboxyhemoglobin levels (usually ≥ 3%-4% for nonsmokers or ≥ 10% for smokers) confirms the diagnosis of CO poisoning and provides information about lactate levels and any concurrent metabolic acidosis. It is important to obtain lactate levels to screen for possible concurrent cyanide toxicity (Lactate > 10 mmol/L) if the source of CO was a fire [18]. While an abnormally elevated COHb level indicates CO poisoning, it is important to note that the COHb levels do not accurately represent the severity of the poisoning. This is particularly true if there has been a significant time lapse between the exposure and when the levels were obtained due to CO clearance. Patients with major symptoms such as loss of consciousness altered mental status, or cardiac ischemia should be considered as severe poisoning with any abnormally elevated COHb level. CO poisoning management should focus primarily on the patient’s signs and symptoms rather than relying solely on the COHb level to guide decision-making.

Pulse oximetry (SpO2), a non-invasive bedside test, cannot be used for screening for CO poisoning, as it doesn’t differentiate oxygenated hemoglobin and carboxyhemoglobin and may yield normal values in CO poisoning despite significant tissue hypoxia. Non-invasive CO oximeters measuring COHb and methemoglobin are available and may have a role as a screening test, but their reliability in clinical settings has been questioned [6]. The American College of Emergency Physicians recommends against using pulse CO oximetry for diagnosis of CO toxicity in patients with suspected acute CO poisoning [2].

An electrocardiogram and a measurement of cardiac enzymes should be included due to the possibility of myocardial injury in patients with moderate to severe CO poisoning looking for myocardial ischemia, infarction, or arrhythmias [2,19]. Imaging studies, such as chest radiographs, may be indicated in certain clinical scenarios and can help patients presenting with hypoxia and dyspnea to evaluate for pulmonary edema [20].

Risk Stratification

Significant neurologic manifestations of CO poisoning include findings such as syncope, coma, seizures, altered mental status (GCS <15) or confusion, and abnormal cerebellar function. Metabolic findings such as lactic acidosis may be profound from cellular hypoxia. Cardiovascular findings include acute myocardial ischemia, myocardial injury, ventricular arrhythmia, and pulmonary edema [6].

The clinical policy from the American College of Emergency Physicians concerning the evaluation and management of adult patients with acute carbon monoxide poisoning presents evidence-based recommendations addressing three key clinical questions: the diagnostic accuracy of noninvasive carboxyhemoglobin measurement, the long-term neurocognitive impact of hyperbaric versus normobaric oxygen therapy, and the predictive value of cardiac testing for morbidity and mortality. The policy is based on a systematic literature review, graded using a defined class of evidence system, and offers recommendations for patient management at varying levels of certainty [21].

According to the ACEP’s CO policy, pulse CO oximetry should not be used to diagnose acute carbon monoxide (CO) poisoning due to its low sensitivity. While it offers advantages like being fast, noninvasive, and cost-effective, studies have shown it detects CO toxicity in only about 48% of cases, meaning it misses half of those affected. Similar findings were reported in other studies.

Both hyperbaric oxygen (HBO₂) and high-flow normobaric oxygen therapies are options for treating acute carbon monoxide (CO) poisoning, but it is unclear if HBO₂ is superior in improving long-term neurocognitive outcomes. While HBO₂ reduces carboxyhemoglobin levels and may aid neurologic recovery, its benefits remain debated. Meta-analyses and studies on HBO₂ have shown inconsistent results, with some finding no benefit and others suggesting improved outcomes. Variations in study designs and treatment factors contribute to the uncertainty, highlighting the need for further research.

In moderate to severe carbon monoxide (CO) poisoning, an electrocardiogram (ECG) and cardiac biomarkers should be used to detect acute myocardial injury, a predictor of poor outcomes. Studies have shown that myocardial injury is associated with higher long-term mortality and is an independent predictor of poor prognosis. Further research is needed to explore cardiac testing and interventions in less severe cases and more aggressive cardiac management for high-risk patients.

Management

Initial management starts with assessing and stabilizing the airway, breathing, and circulation. Comatose patients who have severely impaired mental status or who do not have sufficient respiratory effort should be intubated without delay and mechanically ventilated using 100 percent oxygen [6]. Treatment begins with oxygen therapy, and 100% oxygen should be provided as soon as possible with either a non-rebreather mask or endotracheal intubation, which serves two purposes. First, the half-life of COHb is inversely related to PaO2; it can be reduced from approximately 5 hours in room air to 1 hour by providing supplemental 100% oxygen. HBO therapy (at 3 atmospheres) further reduces the half-life to approximately 30 minutes [12]. Oxygen should be continued until the patient is asymptomatic and carboxyhemoglobin levels are ≤ 3%-4% in nonsmokers and ≤ 10% in smokers [2,18,19]. Evidence suggests that hyperbaric oxygen therapy helps prevent delayed neurologic sequelae in acute CO poisoning, but its efficacy decreases with delayed implementation [15]. HBO therapy can be used in patients presenting with a COHb level >25% (>15% if pregnant), unconscious at scene or hospital, reported syncope, persistent altered, mental status, coma, focal neurologic deficit, severe metabolic acidosis (pH <7.25) after empiric cyanide treatment if administered, or evidence of end-organ ischemia (e.g., ECG changes, elevated cardiac biomarkers, respiratory failure, focal neurologic deficit, or altered mental status). A thorough cardiovascular examination should be performed and should focus on signs of contributing cardiogenic shock or hypotension. Establishing IV access and cardiac monitoring are necessary as patients may need IV fluids or inotropes for resuscitation. An ECG and cardiac enzymes should also be included in the evaluation for cardiac ischemia in symptomatic patients at risk. Patients with altered mental status should have a blood glucose check to evaluate for hypoglycemia [6].

Special Patient Groups

Pediatrics

Children may present with subtle and non-specific findings compared to adults, and it is suggested that they can be more sensitive to the effects of CO due to their higher metabolic rates. Fussiness and decreased oral intake may be the only manifestations of CO toxicity. Although children may have higher levels of COHb due to their higher minute ventilation, which should make them more vulnerable to accumulating CO, the long-term outcomes appear favorable as they have lower rates of developing delayed neurological sequelae compared to adults. The diagnosis and management of CO poisoning in young children generally follow the same principles as for other age groups, with no substantial modifications in approach based on age [6].

Pregnant Patients

There is a lower threshold to using HBO therapy in pregnancy due to the greater affinity and the longer half-life of CO that is bound to fetal hemoglobin, the limited capacity to enhance placental perfusion and the direct effects of acidosis and hypoxemia on the fetus. While severe CO poisoning poses serious short- and long-term fetal risk, mild accidental exposure is likely to result in normal fetal outcomes. Because the fetal accumulation of CO is higher and its elimination slower than in the maternal circulation, hyperbaric oxygen may decrease fetal hypoxia and improve outcomes. While these findings provide valuable insights into the effects of CO poisoning and HBO therapy on pregnant patients and their fetuses, the available literature on this subject remains limited [6].

When To Admit This Patient

Hospitalization is warranted in cases where patients exhibit signs of hemodynamic instability, persistent neurologic symptoms, evidence of end-organ damage (including renal injury, rhabdomyolysis, cardiac ischemia, and pulmonary edema), or exposure to methylene chloride. Most patients who do not meet the criteria for HBO therapy and are not clinically ill can typically be managed in the emergency department; generally, patients who become asymptomatic with a carboxyhemoglobin (COHb) level < 5% may be safely discharged home. All patients exposed to CO require close follow-up for delayed neurologic sequelae [18].

Revisiting Your Patient

Our 48-year-old male, who has a history of prior suicidal attempts, was found unconscious in his home garage with his car engine running. The past medical history and his presentation picture put him at risk for carbon monoxide poisoning, and red flags such as his altered mental state and the recognition of a source of carbon monoxide should guide the clinician through the diagnosis and management process. Management started by assessing the airway, breathing, and circulation. The patient was in a state of respiratory arrest and was intubated and ventilated with 100% oxygen. His pupils were dilated and sluggish. The patient was hypotensive, and IV fluids were started while vasopressors were being prepared. A CBC, chemistry, blood glucose, cardiac enzymes, COHb level, and venous blood gas were requested. A Chest XR was also done, which showed no signs of pulmonary edema, and an endotracheal tube was confirmed in place. ECG showed normal sinus rhythm with no ST-T wave changes. COHb level was 38%, blood glucose 139 mg/dl, and cardiac enzymes were within normal range. His blood gas showed a pH of 7.28 and a lactate of 4. A diagnosis of carbon monoxide poisoning was made. The patient was kept on 100% oxygen and was being prepared to be transferred into a hyperbaric oxygen therapy facility.

Authors

Picture of Mohammad Issa Naser

Mohammad Issa Naser

Dr Mohammad Naser is currently a Critical Care Medicine Fellow in Sheikh Shakhbout Medical City - Abu Dhabi. He completed his emergency medicine training at Zayed Military Hospital and has obtained both the Emirati and Arab board certifications in Emergency Medicine. Dr. Naser has a profound interest in critical care medicine, particularly in bridging the gap between emergency and intensive care practices. Beyond critical care, He is deeply passionate about medical education, mentoring future healthcare professionals, and developing innovative teaching tools. Additionally, he is actively involved in clinical research, focusing on advancing knowledge and practices in emergency and critical care medicine.

Picture of Abdulla Alhmoudi

Abdulla Alhmoudi

Dr Abdulla Alhmoudi is a Consultant Emergency Medicine, serving at Zayed Military Hospital and Sheikh Shakhbout Medical City - Abu Dhabi. He pursued his residency training in Emergency Medicine at George Washington University in Washington DC and further enhanced his expertise with a Fellowship in Extreme Environmental Medicine. Dr Alhmoudi's passion for medical education is evident in his professional pursuits. He currently holds the position of Associate Program Director at ZMH EM program and is a lecturer at Khalifa University College of Medicine and Health Sciences. Beyond medical education, he maintains a keen interest in military medicine and wilderness medicine.

Listen to the chapter

References

  1. Rose JJ, Wang L, Xu Q, et al. Carbon Monoxide Poisoning: Pathogenesis, Management, and Future Directions of Therapy [published correction appears in Am J Respir Crit Care Med. 2017 Aug 1;196 (3):398-399]. Am J Respir Crit Care Med. 2017;195(5):596-606. doi:10.1164/rccm.201606-1275CI
  2. American College of Emergency Physicians Clinical Policies Subcommittee (Writing Committee) on Carbon Monoxide Poisoning:, Wolf SJ, Maloney GE, Shih RD, Shy BD, Brown MD. Clinical Policy: Critical Issues in the Evaluation and Management of Adult Patients Presenting to the Emergency Department With Acute Carbon Monoxide Poisoning. Ann Emerg Med. 2017;69(1):98-107.e6. doi:10.1016/j.annemergmed.2016.11.003
  3. Shin M, Bronstein AC, Glidden E, et al. Morbidity and Mortality of Unintentional Carbon Monoxide Poisoning: United States 2005 to 2018. Ann Emerg Med. 2023;81(3):309-317. doi:10.1016/j.annemergmed.2022.10.011
  4. Rose JJ, Wang L, Xu Q, et al. Carbon Monoxide Poisoning: Pathogenesis, Management, and Future Directions of Therapy [published correction appears in Am J Respir Crit Care Med. 2017 Aug 1;196 (3):398-399]. Am J Respir Crit Care Med. 2017;195(5):596-606. doi:10.1164/rccm.201606-1275CI
  5. Centers for Disease Control and Prevention (CDC). Unintentional non-fire-related carbon monoxide exposures–United States, 2001-2003. MMWR Morb Mortal Wkly Rep. 2005;54(2):36-39.
  6. Manaker S, Perry H. (2023) Carbon monoxide poisoning, UpToDate. Available at: https://www.uptodate.com/contents/carbon-monoxide-poisoning (Accessed: 15 May 2023).
  7. Norris JC, Moore SJ, Hume AS. Synergistic lethality induced by the combination of carbon monoxide and cyanide. Toxicology. 1986;40(2):121-129. doi:10.1016/0300-483x(86)90073-9
  8. Dubrey SW, Chehab O, Ghonim S. Carbon monoxide poisoning: an ancient and frequent cause of accidental death. Br J Hosp Med (Lond). 2015;76(3):159-162. doi:10.12968/hmed.2015.76.3.159
  9. Weaver LK, Howe S, Hopkins R, Chan KJ. Carboxyhemoglobin half-life in carbon monoxide-poisoned patients treated with 100% oxygen at atmospheric pressure. Chest. 2000;117(3):801-808. doi:10.1378/chest.117.3.801
  10. Walker AR. Emergency department management of house fire burns and carbon monoxide poisoning in children. Curr Opin Pediatr. 1996;8(3):239-242. doi:10.1097/00008480-199606000-00009
  11. Rose JJ, Wang L, Xu Q, et al. Carbon Monoxide Poisoning: Pathogenesis, Management, and Future Directions of Therapy [published correction appears in Am J Respir Crit Care Med. 2017 Aug 1;196 (3):398-399].Am J Respir Crit Care Med. 2017;195(5):596-606. doi:10.1164/rccm.201606-1275CI
  12. Meaden CW, Nelson LS. Inhaled Toxins. In: Rosen’s Emergency Medicine Concepts and Clinical Practice. 10th ed. Elsevier; 2023:666-681.
  13. Harper A, Croft-Baker J. Carbon monoxide poisoning: undetected by both patients and their doctors.Age Ageing. 2004;33(2):105-109. doi:10.1093/ageing/afh038
  14. Dolan MC, Haltom TL, Barrows GH, Short CS, Ferriell KM. Carboxyhemoglobin levels in patients with flu-like symptoms. Ann Emerg Med. 1987;16(7):782-786. doi:10.1016/s0196-0644(87)80575-9
  15. Tomaszewski, C. Carbon Monoxide. IN: Goldfrank’s toxicological emergencies. 9th ed. New York: McGraw-Hill Medical Pub. Division; c2011
  16. Cho CH, Chiu NC, Ho CS, Peng CC. Carbon monoxide poisoning in children. Pediatr Neonatol. 2008;49(4):121-125. doi:10.1016/S1875-9572(08)60026-1
  17. Eichhorn L, Thudium M, Jüttner B. The Diagnosis and Treatment of Carbon Monoxide Poisoning.Dtsch Arztebl Int. 2018;115(51-52):863-870. doi:10.3238/arztebl.2018.0863
  18. Hampson NB, Piantadosi CA, Thom SR, Weaver LK. Practice recommendations in the diagnosis, management, and prevention of carbon monoxide poisoning.Am J Respir Crit Care Med. 2012;186(11):1095-1101. doi:10.1164/rccm.201207-1284CI
  19. Weaver LK. Clinical practice. Carbon monoxide poisoning.N Engl J Med. 2009;360(12):1217-1225. doi:10.1056/NEJMcp0808891
  20. Prockop LD, Chichkova RI. Carbon monoxide intoxication: an updated review.J Neurol Sci. 2007;262(1-2):122-130. doi:10.1016/j.jns.2007.06.037
  21. American College of Emergency Physicians Clinical Policies Subcommittee (Writing Committee) on Carbon Monoxide Poisoning:, Wolf SJ, Maloney GE, Shih RD, Shy BD, Brown MD. Clinical Policy: Critical Issues in the Evaluation and Management of Adult Patients Presenting to the Emergency Department With Acute Carbon Monoxide Poisoning. Ann Emerg Med. 2017;69(1):98-107.e6. doi:10.1016/j.annemergmed.2016.11.003

Reviewed and Edited By

Picture of Arif Alper Cevik, MD, FEMAT, FIFEM

Arif Alper Cevik, MD, FEMAT, FIFEM

Prof Cevik is an Emergency Medicine academician at United Arab Emirates University, interested in international emergency medicine, emergency medicine education, medical education, point of care ultrasound and trauma. He is the founder and director of the International Emergency Medicine Education Project – iem-student.org, chair of the International Federation for Emergency Medicine (IFEM) core curriculum and education committee and board member of the Asian Society for Emergency Medicine and Emirati Board of Emergency Medicine.

Meningitis (2024)

by J. Austin Lee

You have a new patient!

A 21-year-old male presented to a clinic. He is a refugee and has been here with a high-grade fever and a severe headache for the past three days. The patient had been working as a laborer in construction sites in the area for the past six months. At triage, his vital signs are as follows: temperature of 39.1°C (102.5°F), blood pressure of 110/70 mmHg, heart rate of 110 beats per minute, and respiratory rate of 22 breaths per minute.

Further examination reveals that the patient is quite photophobic. You note that he prefers to sit still, and when you examine him further, you feel that his neck is quite uncomfortable when flexed, and there is discomfort with flexed hips and passive knee extension. The patient was accompanied by a co-worker who reported that this morning, the patient was vomiting and had been confused. The patient had no history of recent travel or vaccination.

What do you need to know?

Importance

Meningitis is an important infectious disease with severe consequences if not promptly recognized and treated. Meningitis is caused by inflammation of the meninges, the membranes covering the brain and spinal cord. It can be caused by a bacterial, viral, fungal, or parasitic infection. Moreover, meningitis can be triggered by physical injury, autoimmune disorders, cancer, or certain drugs that can cause meningitis. Generally, when discussing meningitis, we are primarily concerned with infectious etiologies. In addition to the high mortality associated with meningitis, survivors may suffer from long-term sequelae, such as hearing loss, cognitive impairment, and neurologic deficits [1]. Infants, children, and immunocompromised patients are at a higher risk of developing meningitis, and outbreaks can occur in crowded living conditions, with classic examples including crowded urban areas (including slums), university dormitories, and military barracks [2]. Prompt recognition and treatment with appropriate antibiotics or antivirals are critical for improving outcomes in patients with meningitis [3].

Epidemiology

Meningitis is a significant global health problem, particularly in low- and middle-income countries. According to the World Health Organization (WHO), there are an estimated 1.2 million cases of bacterial meningitis each year, resulting in 250,000 deaths [4]. According to the Global Burden of Disease study, meningitis is responsible for an estimated 21.9 million disability-adjusted life years (DALYs) globally [5]. The burden of meningitis is particularly high in sub-Saharan Africa, where large-scale epidemics of meningococcal meningitis occur. In these regions, outbreaks are often associated with overcrowding, malnutrition, and poor sanitation, and can cause high rates of mortality and long-term disability. While vaccination has helped to reduce the burden of meningitis in many parts of the world, there is still a need for continued surveillance and control measures, particularly in high-risk populations.

Pathophysiology

Bacteria (and viruses and chemicals) can cross the blood-brain barrier to infect or inflame the meninges by spreading from the bloodstream. Pathogens can also spread from contiguous infection (from a source such as the sinuses or middle ear), trauma, neurosurgery, or indwelling medical devices [6]. Nasopharyngeal colonization from infected droplets of respiratory secretions or distant localized infection (lungs, urine) with subsequent bloodstream invasion are other sources of infection [6].

Once the pathogen reaches the meninges, it triggers an immune response, releasing pro-inflammatory cytokines, which attract immune cells to the site of infection. This immune response leads to the characteristic symptoms of meningitis, including fever, headache, neck stiffness, and altered mental status. In severe cases, the inflammation can lead to increased intracranial pressure, cerebral edema, and brain herniation, which is life-threatening and frequently fatal [6].

Bacterial meningitis poses an emergent risk to the neurological system; progression can result in rapid fatality. Furthermore, bacterial meningitis has the potential to cause long-term complications, including hearing and vision impairment, memory and concentration issues, epilepsy, coordination and balance difficulties, learning challenges, and behavioral disorders [6]. In community-acquired meningitis, S. pneumoniae has become the most common pathogen since routine immunization of infants against H. influenzae type B [7]. It’s important to note that the most common causes of meningitis can vary depending on the patient’s age, geography, and immune status [8]. Table 1 summarizes most common pathogens of meningitis.

Table 1: Common Infectious Causes of Meningitis [7-14].

Pathogen

Common Etiologies

Bacteria

Neisseria meningitidis, Streptococcus pneumoniae, Haemophilus influenzae, Listeria monocytogenes

Viruses

Enteroviruses (e.g. Coxsackie virus, Echovirus), Herpes simplex virus, Varicella-zoster virus, Mumps virus

Fungi

Cryptococcus neoformans, Histoplasma capsulatum, Coccidioides immitis

Parasites

Naegleria fowleri, Acanthamoeba species

Medical History

Key features in the medical history of meningitis include the onset and duration of symptoms, recent travel or exposure to infectious agents, immunization status, underlying medical conditions, and medication use. It is important to obtain a detailed history of present illness, including the timing and progression of symptoms such as fever, headache, neck stiffness, altered mental status, and rash. Patients may also report symptoms such as nausea, vomiting, photophobia, and seizures. Recent travel or exposure to individuals with known or suspected meningitis can help identify potential infectious agents. Immunization status, particularly regarding vaccines against meningococcal and pneumococcal infections, is also important to determine. Patients with chronic medical conditions or who are taking immunosuppressive medications may be at increased risk for certain pathogens or complications.

Physical Examination

The physical exam findings in a patient with meningitis include vital signs, general appearance, and specific neurological findings. Vital signs such as fever, tachycardia, and hypotension are common. Patients may appear acutely ill, with a lethargic or altered mental status. They may exhibit signs of meningeal irritation, such as photophobia, neck stiffness, and a positive Kernig or Brudzinski sign. Kernig’s sign is the inability to straighten the leg when the hip is flexed to 90 degrees; Brudzinski’s sign is positive when forced flexion of the neck elicits a reflex flexion of the hips [6]. Both Kernig and Brudzinski have reported low sensitivity (5%) but high specificity (95%) [6]. Neurological findings such as altered level of consciousness, focal neurologic deficits, and seizures may also occur or be present. Skin findings such as a petechial or purpuric rash may present in meningococcal meningitis patients. In infants, bulging fontanelles and poor feeding are concerning. Jolt accentuation testing can provide additional value: the patient horizontally rotates the head at two to three rotations per second [15]. The worsening of an existing headache indicates a positive result, though the sensitivity of jolt accentuation for diagnosing meningitis varies widely, with estimates ranging from 40-96% [15].

Table 2: Common signs/symptoms of meningitis, with sensitivity [8-10]

Sign / Symptom

Sensitivity

Neck stiffness

30-100

Headache

70-100

Photophobia

50-90

Nausea/vomiting

50-90

Altered mental status

50-80

Jolt accentuation

40-90

Fever

70-80

Seizures

10-30

Focal neurological deficits

<10

Alternative & Differential Diagnoses

  • Encephalitis: inflammation and swelling of the brain parenchyma; encephalitis tends to cause more neurological symptoms such as confusion, seizures, and changes in behavior or personality.
  • Chemical meningitis (e.g., due to contrast agents, medications, or illicit drugs): The patient should have a history of exposure to a triggering agent, such as a medication or contrast dye.
  • Carcinomatous meningitis (e.g., metastatic cancer cells in cerebrospinal fluid); history or imaging with evidence of metastatic disease.
  • Aseptic meningitis (e.g., due to autoimmune disorders, sarcoidosis, or drug reactions) symptoms are usually milder. They may include fever, headache, and neck stiffness, often including other symptoms such as rash or joint pain.
  • Cerebral vasculitis is inflammation and damage to the blood vessels that supply the brain. It may have a more insidious onset and a chronic or recurrent course.
  • Traumatic meningitis (e.g., due to head injury or neurosurgical procedures)
  • Brain abscess or subdural empyema; likely to include more focal neurological symptoms/deficits such as weakness or paralysis, seizures, or speech and vision problems.
  • Subarachnoid bleeding is commonly associated with sudden, severe headaches, nausea, vomiting, and, at times, syncope.
  • Tetanus is commonly associated with other symptoms such as jaw stiffness, diffuse muscle rigidity/spasm, difficulty swallowing, and respiratory distress.
  • Malaria, particularly cerebral malaria, is typically found in areas with high transmission rates of malaria, and cerebral malaria typically has a more gradual onset. It can progress over several days to weeks.

Acing Diagnostic Testing

Acute diagnostic testing is crucial in managing meningitis as it allows for early detection and appropriate treatment. The accepted gold standard for diagnosing meningitis is cerebrospinal fluid (CSF) analysis, obtained through a lumbar puncture [6,16]. CSF analysis includes cell count, protein and glucose levels, culture, and gram stain [16]. Elevated CSF white blood cell count and protein levels are common findings in meningitis, while glucose levels are often decreased. CSF culture and gram stain are essential to identify the causative organism, guide antimicrobial therapy, and can be used to monitor response to treatment.

In addition to CSF analysis, imaging studies such as computed tomography (CT) or magnetic resonance imaging (MRI) may also be obtained to evaluate for complications of meningitis, such as hydrocephalus, cerebral edema, or abscess formation. However, these imaging studies are typically not used for the initial diagnosis of meningitis. CT is a strong consideration to be performed before lumbar puncture (LP) to exclude increased intracranial pressure (ICP) or mass lesion when CT is available and a patient has any of these criteria: immunocompromised state, history of CNS disease, new-onset seizure, papilledema, severe decreased consciousness (GCS<12) or focal neurologic deficit [6].

Blood cultures may also be obtained to help identify the causative organism and determine appropriate antimicrobial therapy. In particular, meningococcemia can rapidly lead to shock and multiorgan failure. Other laboratory tests, such as complete blood count (CBC), chemistry panel, and coagulation studies, are also routinely obtained to evaluate potential complications or comorbidities.

Rapid diagnostic tests, such as polymerase chain reaction (PCR) or antigen tests, may also be available in some settings. These tests can help quickly identify some causes of meningitis, such as bacterial or viral meningitis. They can provide near real-time speciation of the causative organism and help tailor appropriate treatment.

Table 3: CSF Testing Characteristics [9-11, 17-19]

Test

Normal Results

Bacterial Meningitis Results

Viral Meningitis Results

Fungal Meningitis Results

Appearance

Clear, colorless

Cloudy or turbid

Clear to slightly cloudy

Cloudy or turbid

WBC count

<5 cells/microliter

Elevated

Elevated, often lymphocytic

Elevated, often lymphocytic

Glucose

40-70 mg/dL

Decreased

Normal or slightly decreased

Decreased

Protein

15-45 mg/dL

Elevated

Normal to slightly elevated

Elevated

Gram stain

No organisms

Gram-positive or gram-negative organisms

Negative for bacteria, positive for virus

Negative for bacteria and virus

Culture

Negative

Positive for bacterial growth

Negative for bacteria, positive for virus

Positive for fungal growth

Risk Stratification

Several features in the history, physical examination, and testing can indicate a worse outcome in a patient with meningitis. Some of these include advanced age, altered mental status, presence of seizures, hypotension, tachycardia, high cerebrospinal fluid (CSF) protein and low glucose levels, high white blood cell count in CSF, and delayed initiation of appropriate antimicrobial therapy.
Various risk stratification tools have been developed for meningitis, such as the Glasgow Meningococcal Septicemia Prognostic Score (GMSPS), which is used to predict mortality in meningococcal disease. This tool includes variables such as age, Glasgow Coma Scale score, presence of meningismus, and presence of shock. This tool is most helpful in identifying the most sick cases, which are likely to be evident based on the clinical history and exam. Although this score exists, it is not routinely used in clinical practice. Another tool is the Bacterial Meningitis Score (BMS), which helps clinicians differentiate bacterial from aseptic meningitis based on the presence of certain clinical and laboratory features. The BMS includes age, cerebrospinal fluid protein level, cerebrospinal fluid neutrophil count, and peripheral blood absolute neutrophil count.

Management

In patients with whom you have concerns about meningitis, stabilization of an unstable patient is the priority. Assess the airway and breathing, including monitoring the respiratory rate and saturation levels. Administer supplemental oxygen if necessary. Evaluate circulation by checking the pulse, capillary refill time, and blood pressure. Provide fluids or administer medications as required. Next, the neurological function can be evaluated using tools like the Glasgow Coma Scale or AVPU (Alert, Verbal, Painful, Unresponsive) scale. Additionally, glucose levels and the presence of focal neurological signs, seizures, and papilledema should be assessed.

Empiric antibiotics should be started as soon as possible, even before the results of CSF culture and sensitivity are available, in order to reduce the risk of mortality and morbidity. In addition, supportive measures such as fluid and electrolyte management, seizure prophylaxis, and management of increased intracranial pressure are essential in managing meningitis. Patients with severe disease or complications may require ICU admission. Close follow-up with repeat CSF analysis and neuroimaging may be necessary to monitor response to treatment and identify potential complications.

Empiric treatment for bacterial meningitis typically involves using third-generation cephalosporins, such as ceftriaxone or cefotaxime, with or without vancomycin to cover for potential penicillin-resistant strains of Streptococcus pneumoniae. In infants under 1 month of age and patients over 50 years, ampicillin is often added to cover for Listeria monocytogenes [2]. Dexamethasone, a corticosteroid, is also given prior to or at the time of antibiotic initiation in adults and children with suspected or confirmed bacterial meningitis to reduce the risk of neurologic sequelae. The administration of corticosteroids has been shown to significantly reduce hearing loss and neurological complications in patients with meningitis.

However, using corticosteroids has not significantly impacted overall mortality rates [20]. The management of viral meningitis is mainly supportive. Antiviral treatment may be considered for specific viral pathogens, such as acyclovir for herpes simplex virus (HSV) or ganciclovir for cytomegalovirus (CMV). However, empiric antiviral treatment is not recommended in most cases of viral meningitis. The use of corticosteroids, such as dexamethasone, is controversial in viral meningitis and is not generally recommended [20].
Pre-exposure prophylaxis, though intrapartum prophylaxis of group B streptococcus in pregnant women, has significantly reduced the risk of early-onset group B strep meningitis [21]. Post-exposure prophylaxis is also an important consideration in contacts of patients diagnosed with meningitis; close contacts are defined as individuals who have had prolonged close contact with the index case, such as household contacts, healthcare workers, or individuals who shared a room or had direct contact with respiratory or oral secretions. Antibiotic prophylaxis is typically recommended within 24-48 hours of identification of the index case and may include rifampin, ciprofloxacin, or ceftriaxone, depending on the age and health status of the contact. In addition to antibiotics, vaccination with the meningococcal conjugate vaccine may be recommended for close contacts, particularly those at increased risk.

The recommended antibiotic prophylaxis is usually a single dose of intramuscular ceftriaxone (250 mg for adults and children weighing > 45 kg and 125 mg for children weighing < 45 kg). Alternatively, oral antibiotics such as rifampin, ciprofloxacin, or azithromycin can be used as alternatives. For exposure to Streptococcus pneumoniae, oral amoxicillin is recommended for prophylaxis, and for exposure to Haemophilus influenzae type b (Hib), rifampin or ceftriaxone is recommended.

Special Patient Groups

Elderly individuals, particularly those over 65, may present with atypical meningitis characterized by lethargy, minimal signs of meningismus, and the absence of fever. Conversely, younger individuals such as neonates, infants, and children often present with symptoms such as poor feeding, irritability, fever, and in babies, a shrill cry, decreased appetite, rash, and vomiting. In young children, the presentation of meningitis can mimic flu-like symptoms, including cough or respiratory distress, and it is not uncommon for them to have a history of respiratory tract infection. Seizures are also more frequently observed in this age group with meningitis. When evaluating a febrile child who appears unwell, it is crucial to consider bacterial meningitis as a potential diagnosis until ruled out. It is worth noting that blood and cerebrospinal fluid results may appear normal, especially in extremely young or old age groups.

When To Admit This Patient

Patients with suspected meningitis should be admitted to the hospital from the emergency department, as this is a potentially life-threatening condition that requires urgent evaluation and treatment. Admission should be considered for patients with a high likelihood of meningitis based on clinical presentation and laboratory findings. Patients with severe symptoms such as altered mental status, seizures, or signs of sepsis are particularly high-risk and should be admitted promptly. Patients with risk factors such as immunocompromised status, recent head trauma, or history of neurosurgical procedures should also be admitted.

Patients with meningitis who present with severe symptoms or complications such as altered mental status, seizures, respiratory distress, or signs of sepsis should be considered for admission to the intensive care unit (ICU). In addition, patients with bacterial meningitis or other severe forms of meningitis, such as fungal or tuberculous meningitis, and those immunocompromised should also be admitted to the ICU for close monitoring and aggressive treatment. Patients with a high risk of developing cerebral edema or increased intracranial pressure, such as those with hydrocephalus or brain abscess, may also require ICU admission. Close monitoring of vital signs, neurologic status, and laboratory parameters, such as blood glucose and electrolytes, is likely best done in an ICU.

Revisiting Your Patient

Let’s go back to the clinical presentation of your 21-year-old male refugee. He has fever, tachycardia, vomiting and confusion, and meningitis was suspected. You performed a lumbar puncture, and the cerebrospinal fluid analysis showed a white cell count of 1500 cells/µL with predominant neutrophils, protein level of 150 mg/dL, and glucose level of 30 mg/dL. The patient was started on treatment with intravenous ceftriaxone and vancomycin and admitted to the hospital. The patient was diagnosed with bacterial meningitis and was continued on intravenous antibiotics for a total of 14 days.

The patient responded well to the treatment and was discharged after completing the course of antibiotics. Appropriate public health notification was made, and the patient was scheduled for post-discharge follow-up care and vaccination.

Author

Picture of J. Austin Lee, MD MPH DTMH

J. Austin Lee, MD MPH DTMH

Austin Lee, MD MPH DTMH, is a practicing emergency medicine doctor in the United States. He currently works with Indiana University Health, across several hospital sites. Dr. Lee obtained an MPH at the George Washington University before going to medical school at Indiana University. He completed his emergency medicine residency at the University of Virginia, and then worked at Brown University where he was a part of the Global Emergency Medicine fellowship. Austin has worked on a number of international emergency medicine projects, and is actively engaged in supporting the development of emergency medicine in Kenya.

Listen to the chapter

References

  1. Tunkel AR, Scheld WM. Acute meningitis. In: Jameson JL, Fauci AS, Kasper DL, Hauser SL, Longo DL, Loscalzo J, eds. Harrison’s Principles of Internal Medicine. 20th ed. Vol 1. New York, NY: McGraw-Hill Education; 2019:894-900.
  2. Mandell GL, Bennett JE, Dolin R. Principles and Practice of Infectious Diseases. 8th ed. Vol 2. Philadelphia, PA: Elsevier; 2015:1116-1132.
  3. Longo DL, Kasper DL. Bacterial meningitis. In: Longo DL, ed. Harrison’s Infectious Diseases. 3rd ed. New York, NY: McGraw-Hill Education; 2018:360-373.
  4. World Health Organization. Defeating meningitis by 2030. Accessed May 25, 2023. https://www.who.int/initiatives/defeating-meningitis-by-2030.
  5. GBD 2016 Meningitis Collaborators. Global, regional, and national burden of meningitis, 1990-2016: a systematic analysis for the Global Burden of Disease Study 2016. Lancet Neurol. 2018 Dec;17(12):1061-1082.
  6. Parežnik A. Meningitis. October 12, 2018. Accessed May 25, 2023. https://iem-student.org/meningitis/.
  7. McGill F, Heyderman RS, Panagiotou S, Tunkel AR, Solomon T, Connor MD. Acute bacterial meningitis in adults. The Lancet. 2016;388(10063):3036-3047.
  8. Lu CH, Chang WN, Chang HW, et al. Adult bacterial meningitis in southern Taiwan: epidemiological trend and prognostic factors. J Neurol Sci. 2005;22(2):133-139.
  9. van de Beek D, de Gans J, Spanjaard L, et al. Clinical features and prognostic factors in adults with bacterial meningitis. N Engl J Med. 2004;351(18):1849-1859.
  10. Tunkel AR, Hartman BJ, Kaplan SL, et al. Practice guidelines for the management of bacterial meningitis. Clin Infect Dis. 2004;39(9):1267-1284.
  11. van de Beek D, de Gans J, Tunkel AR, et al. Community-acquired bacterial meningitis in adults. N Engl J Med. 2006;354(1):44-53.
  12. McGill F, Griffiths MJ, Solomon T. Viral meningitis: current issues in diagnosis and treatment. Curr Opin Infect Dis. 2017 Apr;30(2):248-256.
  13. Góralska K, Blaszkowska J, Dzikowiec M. Neuroinfections caused by fungi. Infection. 2018 Aug;46(4):443-459.
  14. Pana A, Vijayan V, Anilkumar AC. Amebic Meningoencephalitis. 2023 Jan 21. In: StatPearls [Internet]. Treasure Island (FL): StatPearls Publishing; 2023 Jan
  15. Iguchi M, Noguchi Y, Yamamoto S, Tanaka Y, Tsujimoto H. Diagnostic test accuracy of jolt accentuation for headache in acute meningitis in the emergency setting. Cochrane Database Syst Rev. 2020 Jun 11;6(6):CD012824.
  16. Roberts JR, Custalow CB, Thomsen TW. Roberts and Hedges’ Clinical Procedures in Emergency Medicine and Acute Care. Elsevier; 2018.
  17. Kumar R, Bose M, Singh SN, et al. Clinicoradiological and neurophysiological correlation in Japanese encephalitis. Ann Trop Paediatr. 1994;14(4):311-318.
  18. González-Duarte A, Cárdenas G, Torres-Narbona M, et al. Cerebrospinal fluid lactic acidosis in aspergillosis meningitis. Arch Neurol. 2007;64(9):1362-1364.
  19. Brouwer MC, Tunkel AR, van de Beek D. Epidemiology, diagnosis, and antimicrobial treatment of acute bacterial meningitis. Clin Microbiol Rev. 2010;23(3):467-492.
  20. van de Beek, D., de Gans, J., McIntyre, P., Prasad, K., & Weisfelt, M. (2004). Corticosteroids for acute bacterial meningitis. The Cochrane Database of Systematic Reviews, (1), CD004405.
  21. Thigpen MC, Whitney CG, Messonnier NE, Zell ER, Lynfield R, Hadler JL, et al. Bacterial meningitis in the United States, 1998-2007. N Engl J Med. 2011;364(21):2016-2025.

Reviewed and Edited By

Picture of Arif Alper Cevik, MD, FEMAT, FIFEM

Arif Alper Cevik, MD, FEMAT, FIFEM

Prof Cevik is an Emergency Medicine academician at United Arab Emirates University, interested in international emergency medicine, emergency medicine education, medical education, point of care ultrasound and trauma. He is the founder and director of the International Emergency Medicine Education Project – iem-student.org, chair of the International Federation for Emergency Medicine (IFEM) core curriculum and education committee and board member of the Asian Society for Emergency Medicine and Emirati Board of Emergency Medicine.

Delirium and Dementia (2024)

by Lo Lucian Simeon, Ngai Oona Wing Yan, & Lo Yat Hei

You have a new patient!

Adam is a 76-year-old man who is brought to the emergency room by his family members, complaining of a lack of responsiveness and general lethargy. According to his family, Adam has been having increasing memory problems in the past year and has gotten lost while walking around his neighborhood multiple times. His personality has changed and becomes agitated easily. He is also becoming less attentive to personal hygiene, wearing dirty clothes for several days, and having several episodes of urinary incontinence. Today, his family members noted that he had fallen asleep multiple times and showed no interest in his food. He did not respond when addressed by name. At the time of presentation, he is conscious, but appears lethargic and uncooperative. He cannot tell where he is and does not seem to recognize his family members. His past medical history includes hypertension and hypercholesterolemia. He is taking amlodipine and simvastatin.

Vitals show a heart rate of 108 beats per minute, blood pressure 154/84 mmHg, temperature 36.7℃, respiratory rate 20 breaths per minute, and an oxygen saturation of 98% on room air. His Glasgow coma score is E4V4M6.

What do you need to know?

Importance

Dementia and delirium are two medical conditions that significantly impact the health and well-being of older adults and their families. In this case, Adam’s symptoms suggest that he may be experiencing one or both of these conditions, and it is important to understand their relevance in clinical practice.

Dementia is defined as an acquired global decline in cognitive function, affecting one’s memory, language, learning, and behavior without impairment of consciousness. Dementia is associated with a gradual, progressive decline. It is a leading cause of disability and dependence among older adults, with advancing age being one of the most significant risk factors [1]. With the global population aging, the number of individuals living with dementia is expected to rise significantly. The World Health Organisation estimates that 47 million people worldwide live with dementia, and this number is expected to triple by 2025 [2].

Delirium, conversely, is a clinical syndrome characterized by an acute state of confusion, inattention, and cognitive impairment. It can occur in people of any age, but is particularly common among the older population and hospitalized patients [1]. Delirium can wax and wane over time, unlike dementia, which is more progressive and persistent.

Dementia and delirium pose a tremendous burden not only on patients and caregivers, but also on our healthcare system and society. Therefore, understanding the significance of dementia and delirium is crucial in clinical practice. Identifying and managing these conditions early improves clinical outcomes and optimizes quality of life.

Epidemiology & Pathophysiology

Dementia is a condition that is more commonly seen in older individuals, with the incidence increasing from the age of 65.  An exception is frontotemporal dementia, a rare type of dementia that is usually diagnosed from the age of 40 to 60. The most prevalent type of dementia is Alzheimer’s disease, which accounts for 60-80% of all cases. Other neurodegenerative dementias, such as vascular dementia, dementia due to Lewy bodies, Parkinson’s disease, and frontotemporal dementia, account for the remaining cases [3].

The underlying pathophysiology of dementia varies depending on the type and subtype, with most types involving damage to neurons and their connections in the brain. Abnormal protein accumulation is a common feature for many types of dementia, including amyloid and tau in Alzheimer’s disease, Lewy bodies with alpha-synuclein protein in Lewy body disease, and mutations causing the deposition of TDP-43 and tau proteins in frontotemporal dementia. Other factors, such as ischemic injury, HIV infection, and alcohol consumption, can also lead to cytotoxic processes in the brain and contribute to the development of dementia [4].

On the other hand, the epidemiology of delirium is more complex as it varies depending on age and underlying medical conditions. Although delirium is more prevalent in older individuals, with rates increasing after the age of 70, it is also common in younger patients suffering from chronic illnesses such as cardiovascular and renal comorbidities, dementia, or psychiatric illnesses [5].

The pathophysiology of delirium can result from various physiological and structural lesions in the brain. While its mechanisms are not fully understood, delirium can be caused by neurotransmitter imbalances, brain lesions involving the ascending reticular activating system, as well as disrupted blood-brain barrier function that causes the leakage of neurotoxic agents into the brain. Patients with impaired cholinergic transmission, such as those with Alzheimer’s disease, are particularly susceptible to delirium caused by medication use. Additionally, delirium can result from alcohol abuse, drug withdrawal, mental illnesses, psychosocial stress, and sleep deprivation [6].

Epidemiological and pathophysiological data on dementia and delirium allow physicians to identify individuals at risk and intervene appropriately. Since the development of dementia and delirium are multifactorial and the pathophysiology is variable among patients, evaluating and treating delirium and dementia is based on clinical gestalt and the presumed underlying cause.

Medical History

Dementia and delirium are two diagnoses that must be considered in elderly patients presenting with cognitive change to the emergency department. Differentiation between the two conditions is based on features noted in the history and physical examination. Table 1 lists symptoms that can help differentiate between the two conditions.

Delirium typically presents with sudden onset of impaired awareness, confusion, clouding of consciousness, and disturbances of perception (e.g., illusions or hallucinations). Delirium should be suspected when there is an acute deterioration in behavior, cognition, and daily functioning [7]. Delirious patients usually have short-term memory issues and may be disoriented by time and place. Abnormalities of cognition and behavior can fluctuate over brief periods. The level of awareness may range from hypervigilant and agitated to blunt and unreactive. The patient’s speech may be incoherent, nonsensical, or tense. The patient usually has no discernible focal neurological defect [8].

Dementia has various presentations according to the specific types, but symptoms often overlap. Alzheimer’s dementia, the most common type, presents with a history of a chronic, steady decline in cognitive ability, especially memory. It is often associated with difficulties in social relationships, activities of daily living, and work. During the early stages of dementia, clinical presentations can be quite subtle, and patients may try to hide their cognitive impairments [9].

Patients who present to the emergency department with symptoms of dementia are most likely in the later stages of disease progression. Acute presentation of dementia is possible in vascular dementia, and this subtype may present with symptoms of focal neurological deficit. Patients who are demented typically do not present with any impairment of consciousness. However, acute episodes of delirium can be superimposed on patients who have dementia.  For example, Lewy body dementia can present with fluctuating levels of consciousness. Diagnosing uncommon variants poses a challenge to emergency physicians and is often done only after referral to a neurologist [10].

Obtaining a thorough history is essential in diagnosing delirium and dementia. Unfortunately, delirious or demented patients are often disoriented and cognitively impaired, resulting in the inability to provide accurate information about their condition. A detailed history should be obtained from family, caregivers, and healthcare staff (nurses, healthcare assistants, and other allied health professionals).

Important features to note during history are the onset of symptoms, factors that worsen or improve symptoms, drug or alcohol use, pre-existing endocrine or psychiatric disorders, exposure to toxins or traumatic injury, social history, and previous similar episodes of confusion or altered mental status. Drug history is particularly important as the use of drugs that impair cognition (e.g., analgesics, anticholinergics, psychotropic medications, and sedatives) may explain the presenting symptoms. 

Determining the onset of symptoms, in particular, for patients with dementia, can be difficult due to the gradual nature of the disease. Questions like, “When did you first notice the memory loss?” and “How has the memory loss progressed since then?” can give a general idea on the patient’s current condition. The patient’s social history, especially work, educational history, and ability to conduct activities of daily living, can help establish a baseline for the patient.

Table 1: Key symptoms to look for to differentiate between delirium and dementia during history taking [11]

 

Delirium

Dementia

Onset

Acute

 Insidious/chronic

Course

 

 Fluctuating

Progressive

Duration

 Days to weeks

 Months to years

Consciousness

 

 Altered

Clear

Alertness

 Impaired

Normal, except for in severe cases of dementia

Behaviour & Speech

 

Agitated/withdrawn/ depressed/combination of symptoms

Intact early on

Typical presentations of dementia of various type

Dementia manifests in various forms, each with distinct characteristic presentations. Alzheimer’s dementia typically involves memory loss, mood instability, apathy, and may include depressive or paranoid features. Additionally, patients may experience apraxia, anosognosia, sensory inattention, and progressive personality and intellectual deterioration. Vascular dementia, on the other hand, often has an abrupt onset with a stepwise deterioration and a fluctuating course. It is marked by slowed thinking, difficulties in organization, preserved personality and insight, and may include focal neurological deficits. Dementia with Lewy bodies is characterized by Parkinsonism, cognitive and alertness fluctuations, as well as visual hallucinations, delusions, and autonomic dysregulation. Frontotemporal dementia commonly occurs at a younger age, typically between 40 and 60 years, and is associated with early personality changes, disinhibition, and overactivity.

Physical Examination

The physical exam of the dementia and delirium patient starts with taking vital signs, assessing the airway, breathing, circulation, and performing a focused neurological exam.  Calculating the Glasgow Coma Score (GCS) and checking blood glucose should be checked on all patients with behavioral or cognitive changes.

Table 2 lists the key signs of differentiating delirium and dementia. Key features such as acute onset, fluctuations in awareness, orientation, and consciousness, cognitive decline, and potential sensory disruptions can help distinguish delirium from dementia. This includes declining memory function, language ability, and judgment. When in doubt, the general rule of thumb is to assume the patient is having an episode of delirium and try to rule out the common causes. This rule can be applied even for patients with known psychiatric illnesses like depression and dementia, as they are also susceptible to delirium superimposed on their existing condition.

Table 2: Key signs to look for to differentiate between delirium and dementia during physical exam [11]

 

Delirium

Dementia

Conscious level

Abnormal

 Normal

Psychomotor changes

Increased/decreased

Often normal

Reversibility of symptoms

Reversible usually

 Irreversible

On neurological examination, look for signs of stroke, parkinsonism, gait abnormalities, and abnormal eye movements. Dementia caused by Alzheimer’s disease generally has no sensory or motor deficits. Whereas for delirium, it is essential to identify any co-existing neurological disorders that may cause a presentation of delirium. Special tests for gait, daily living, and cognitive function assessment should be done to assess the severity of the patient’s condition. A thorough physical examination of other systems should also be conducted to look for signs of encephalopathy and drug and alcohol abuse.

Use the physical exam to help identify any exacerbations of an underlying medical illness (e.g., signs of diabetic ketoacidosis in a diabetic patient) and to evaluate for signs that may reveal an underlying cause.  For example, a high fever, low blood pressure, rapid or slow heart rate, difficulty breathing, severe pain, or malaise may indicate delirium caused by an infection, sepsis, or shock that requires immediate medical attention. Severe thirst, nausea, and vomiting may indicate dehydration or electrolyte disturbances that should be promptly treated. Signs of unresponsiveness, difficulty breathing, or seizures may indicate intracranial bleeding or alcohol or drug intoxication. Since these conditions can cause delirium and have symptoms that overlap with dementia, it is important to prioritize and appropriately manage these urgent and life-threatening cases [12].

Alternative Diagnoses

Table 3 shows alternative diagnoses to consider when evaluating for dementia and delirium. In patients presenting with altered cognitive levels, life-threatening causes that need to be ruled out ​​include hypoglycemia, electrolyte abnormalities such as hyponatremia and hyperkalemia, dehydration, stroke, intoxication/overdose, encephalopathy, cerebral infection, sepsis, and shock.

Psychiatric disorders such as psychosis, schizophrenia, and depression are among the list of differential diagnoses that could present with similar symptoms. Frequently, in patients with delirium, they do not have any previous history of psychiatric illness. In delirium, hallucinations and illusions are acute or subacute and fluctuate over time. In addition, the patient has impaired memory, orientation, and judgment, as well as clouding of consciousness. Elderly patients with a depressed mood, hopelessness, and suicidal ideation may be suffering from “pseudodementia” (false dementia). When the symptoms of depression are treated, the dementia-like condition usually resolves itself [13].

Investigations, such as bloodwork, toxicology screening, biochemical tests, and imaging can help determine a delirium patient’s underlying cause and identify an alternative diagnosis.  Investigations to consider are listed below under “Acing diagnostic testing.”

Table 3: Alternative diagnoses of altered cognitive level [14]

Central nervous system: brain abscess, cerebral neoplasm, encephalitis, intracranial haemorrhage, meningitis, normal pressure hydrocephalus, variant Creutzfeldt-Jakob Disease and bovine spongiform encephalopathy

Electrolyte: hyper/hypocalcemia, hyperkalemia, hyper/hyponatremia

Dehydration

Environmental: heat stroke, snake bite

Infective: sepsis, rabies, malaria

Metabolic: diabetic ketoacidosis, hyperosmolar hyperglycemic nonketotic coma, hypoglycemia, hypothyroidism, uremia, hepatic encephalopathy

Nutrition deficiency: folate, thiamine (Wernicke encephalopathy), vitamin B12,

Poisoning: amphetamine, anticholinergic, antidepressant, cocaine, hallucinogen, lithium, tricyclic antidepressant, valproate, withdrawal

Psychiatric: depression, psychosis

Transient global amnesia

Acing Diagnostic Testing

Initial Investigations

  • Complete Blood Count with Differential: This test is critical for assessing overall health and detecting a variety of conditions, such as infections, anemia, and blood disorders. The differential component provides a breakdown of different types of white blood cells, which can help to identify specific types of infections (e.g., bacterial or viral) and help diagnose other hematological disorders like leukemias or other abnormalities in blood cell production.

  • Electrolyte Panel: The electrolyte panel is essential for assessing the balance of minerals in the body, such as sodium, potassium, calcium, and chloride. Disturbances in these levels can indicate a variety of issues. For instance, hyponatremia (low sodium) can be a sign of dehydration or kidney dysfunction, while hyperkalemia (high potassium) could indicate kidney failure or metabolic acidosis. These imbalances can have significant effects on muscle function, nerve transmission, and overall cellular processes.

  • Liver Function Test: Liver function tests are crucial in diagnosing liver diseases such as hepatitis, cirrhosis, and alcoholic liver disease, as well as conditions like hepatic encephalopathy. These tests measure the levels of enzymes, proteins, and substances like bilirubin, which indicate how well the liver is working. Abnormal results may suggest liver damage, bile duct obstruction, or liver dysfunction that can lead to brain symptoms, especially in severe cases of hepatic encephalopathy.

  • Renal Function Test: This test evaluates how well the kidneys are filtering waste from the blood. It includes measurements like serum creatinine and blood urea nitrogen (BUN), which are key indicators of kidney function. Elevated levels may suggest renal failure, and abnormalities in these values can also help diagnose uremia, a condition where kidney dysfunction leads to the accumulation of waste products in the blood, potentially affecting multiple organ systems.

  • Blood Sugar: Blood sugar levels are measured to rule out conditions like hypoglycemia (low blood sugar) and hyperglycemia (high blood sugar). In patients with diabetes, particularly in cases of diabetic ketoacidosis (DKA) or hyperosmolar hyperglycemic nonketotic syndrome (HHNS), these levels can be critically elevated and require immediate treatment. Monitoring blood sugar is essential for managing and preventing complications related to these conditions.

  • Urine Dipstick: The urine dipstick test is a quick and convenient method for identifying potential urinary tract infections (UTIs), which are a common cause of sepsis in elderly patients. It can detect substances like white blood cells, nitrites, and protein in the urine, all of which suggest infection or inflammation. Early detection of UTIs is crucial, as they can quickly progress to sepsis if untreated.

  • Chest X-ray: A chest X-ray is an important imaging tool for identifying lung consolidation, a hallmark of chest infections such as pneumonia. Pneumonia is another common cause of sepsis, particularly in elderly patients with weakened immune systems. The X-ray can also help detect other lung-related issues like fluid accumulation, pulmonary edema, or lung tumors that could complicate the clinical picture.

  • CT Scan of the Brain: A CT scan of the brain is used to identify structural abnormalities, including the presence of tumors, stroke, or brain hemorrhages. It is also used to detect cerebral atrophy (shrinkage of brain tissue) and ventricular enlargement, which can be indicative of conditions like dementia. This imaging modality is important in diagnosing neurological disorders and guiding further management for patients with cognitive or neurological impairments.

Further Investigations If A Differential Is Suspected

  • Urine and Blood Toxicology: This test is performed to detect the presence of drugs, alcohol, or other toxic substances in the body. Toxicology screens can identify intentional or unintentional overdoses, exposure to toxic substances, and drug or alcohol misuse. In cases of altered mental status or cognitive impairment, toxicology testing helps to rule out substance-induced confusion or delirium, which can mimic other medical or psychiatric conditions.

  • Thyroid Function Test: Thyroid hormones play a significant role in regulating metabolism and overall brain function. Abnormal thyroid function, whether hypothyroidism (underactive thyroid) or hyperthyroidism (overactive thyroid), can lead to symptoms of cognitive impairment, mood changes, and lethargy. A thyroid function test measures levels of thyroid hormones (such as TSH, T3, and T4) to determine if an imbalance is contributing to the patient’s cognitive or neurological symptoms, which can be reversible with appropriate treatment.

  • Vitamin B12 and Folate Levels: Both Vitamin B12 and folate are essential for nerve function and the production of red blood cells. A deficiency in either of these vitamins can lead to cognitive impairment, memory loss, and other neurological symptoms. Vitamin B12 deficiency, in particular, is known to cause a condition called subacute combined degeneration of the spinal cord and brain, which can lead to irreversible damage if left untreated. Checking these levels helps to rule out nutritional deficiencies as a potentially treatable cause of cognitive decline.

  • Bacteriology and Viral Detection: Infection-related causes of cognitive impairment or altered mental status may be identified through bacteriology and virology testing. This typically involves blood and urine microscopy, as well as culture tests to detect bacterial, viral, or other pathogenic organisms. Infections, especially in elderly or immunocompromised patients, can lead to sepsis or encephalitis, which can significantly impact cognitive function. Identifying and treating an underlying infection can prevent further deterioration and improve cognitive outcomes.

Risk Stratification

Cognitive assessment tools help identify and grade abnormal cognitive performances. They can also be integrated into the emergency medicine physical exam to screen patients for mild cognitive impairment or dementia.

Instead of an extended mental status examination or formal neuropsychological testing, more focused screening tools are more relevant and feasible for use in the emergency department [15]. Examples of screening tools validated for use in the emergency department include the abbreviated mental test score (AMTS) and its abbreviated four-item version (AMT4), the mini-mental state examination (MMSE), and the Montreal Cognitive Assessment (MoCA). They assess a broad range of cognitive domains, including memory, language, and orientation. These tests are designed to be administered in 15 minutes or less and have pre-determined cut-off scores to help distinguish patients with normal cognitive function and those with impaired cognitive function [16].

These screening tools are intended to help screen individuals who may require more extensive neurological assessments. They should only be used as a reference and must be integrated with history and physical examination findings for a holistic approach. Multiple factors, such as altered mood, disorientation, and education level, can affect the accuracy of these screening tools.

Table 4: Abbreviated mental test score four-item version (AMT4)

Ask the patient to state each of the following. A score less than 4 should prompt further cognitive screening.

Age

Correct (+1) / Incorrect (0)

Date of birth

Correct (+1) / Incorrect (0)

Place

Correct (+1) / Incorrect (0)

Year

Correct (+1) / Incorrect (0)

Management

The ABCDE approach is used for the initial management of patients with cognitive changes, behavioral changes, and alterations in consciousness, which may be present in delirium or dementia.

The acute management of dementia and delirium is variable and depends on the patient’s underlying medical conditions and presenting symptoms. The main goal of managing dementia and delirium in the emergency department is to identify and treat any life-threatening underlying causes. Based on the patient’s signs and symptoms, a thorough history, physical exam, and pertinent investigations should be ordered. Patient and staff safety should also be prioritized, as these patients may be aggressive and combative.

Obtaining investigations to evaluate for the underlying cause may be hindered by the patient’s aggressive and combative state. If this is the case, the first attempt is to calm the patient and de-escalate the situation verbally.  If unsuccessful, chemical sedation should be considered for the safety of the patient and the healthcare staff. Examples of chemical sedation used in an emergency department setting include benzodiazepines, antipsychotics, and dexmedetomidine [17,18]. Close monitoring is necessary after the patient is sedated. Sedatives should be used only when necessary, as they have the potential to worsen delirium and disorientation.

Physical restraints and environmental seclusion are other adjunctive treatments for agitated delirium or dementia patients. However, their use should be weighed with the psychological and physical risks they may cause (e.g., emotional distress, skin and soft tissue injuries, orthopedic injuries, rhabdomyolysis, etc.). Alternative methods of managing agitation should always be attempted prior to physical restraint, such as explaining your desire to care for the patient, orienting the confused patient to his or her surroundings, using verbal de-escalation techniques, providing psychosocial support, and relocating to a calm and quiet environment, if possible [19].

Special Patient Groups

Most patients presenting with delirium and/or dementia are elderly patients. Younger patients (<60 years) presenting with delirium or patients who have rapidly progressing dementia may require extensive evaluation to discover the underlying cause. Further investigations could include lumbar puncture, electroencephalography, advanced neuroimaging, neuropsychological, and genetic testing [20]. Regardless of age, the most common causes of early-onset dementia are still Alzheimer’s disease, vascular dementia, and frontotemporal dementia [21].

When To Admit This Patient

Admission of dementia and delirious patients depends on various factors, including the severity of symptoms, comorbidities, and safety concerns. Patients who present with acute changes in their mental status, such as sudden confusion or agitation, should be further assessed for any underlying medical conditions and often require hospitalization. Delirious patients, particularly those with severe symptoms or who are at risk of harming themselves and others, should also be admitted until stabilized. Ultimately, the decision to admit dementia and delirious patients in an emergency department setting should be based on a comprehensive evaluation of the patient’s medical history, current symptoms, and risk factors.

The patient who is coming to the emergency department for a chronic presentation of Alzheimer’s dementia could be discharged if life-threatening conditions have been ruled out and home safety is not a concern. They should be referred to an outpatient primary care doctor or a geriatrician for follow-up and prescription medications to manage behavioral symptoms. Be sure to educate the patient’s family members on the diagnosis and to monitor for any new or worsening symptoms that may require urgent medical attention. Advise the family on managing certain scenarios, prioritizing the patient’s basic daily needs, addressing any medical concerns, and maintaining patient and family safety. Refer the patient to a geriatric community support program, if available.

Revisiting Your Patient

Adam’s initial vital signs are stable, and you have decided to continue his management in the consultation room. His history of cognitive and behavioral change over the past year is consistent with dementia. However, his acute presentation of impaired consciousness level and disorientation raises your suspicion of concurrent delirium from an underlying medical condition. Collateral history from the family indicates that Adam lives with his wife, who is 85 years old and limited in her ability to assist Adam with his daily needs. Focused drug and alcohol history is unrevealing.

Further neurological exams do not identify any focal neurological signs or gait disturbance. Physical examinations of the cardiovascular, respiratory, and abdomen are unremarkable. Blood glucose is within the normal range. Due to his disorientation, his Glasgow coma score is 14 (E4/V4/M6).

You consider using AMT4 to screen for cognitive impairment. He can recall his age and date of birth and tell where he is, but he fails to tell us the current year. You establish that he has delirium with impaired alertness and likely an underlying cognitive impairment.

You decide to conduct further investigation to look for potential underlying causes, especially those which may prompt immediate treatment. You arrange blood tests, including complete blood count, electrolytes, liver, and renal function tests, in consideration of potential sepsis, electrolyte disturbance, and acute organ failure. You arrange a CT brain to rule out any acute cerebral hemorrhage and space-occupying lesion. Chest X-ray and ECG are performed, as well. As you order these investigations, you consider that the patient may become agitated and uncooperative during these tests, so you review options for chemical sedation should they be needed.

You discuss openly with Adam and his family on his diagnosis of delirium and likely dementia. Your preliminary investigations show a urinary tract infection; one dose of intravenous antibiotics has been ordered. You suggest admitting Adam for monitoring and investigations due to safety concerns and his inability to care for himself due to his recent rapid decline.  You discuss the importance of appropriate follow-up care and geriatric resources specializing in dementia. The patient is admitted to the general medical floor for further testing and monitoring.

Authors

Picture of Lo Lucian Simeon

Lo Lucian Simeon

Lucian Lo is a medical student at The Chinese University of Hong Kong. An avid enthusiast of emergency medicine and humanitarian work, he hopes to one day combine his two great passions as a front-line healthcare professional in conflict and disaster zones. He is a certified Advanced Medical Life Support Provider and Youth Mental Health First Aid Provider. In addition, he has led and organized multiple medical service projects in Hong Kong, Nepal, and Thailand. In regard to emergency medicine, his interests include trauma care, intensive care medicine, and pre-hospital emergency medicine.

Picture of Ngai Oona Wing Yan

Ngai Oona Wing Yan

Oona Ngai is a medical student at The Chinese University of Hong Kong with a passion for emergency medicine and humanitarian work. She has organized and participated in various volunteer services for vulnerable communities in Hong Kong, including the homeless, refugees, and domestic helpers. Oona is also a St. John’s certified Advanced Medical Life Support Provider and aspires to better equip herself with the necessary skills and knowledge to provide effective medical care in emergency situations. In addition, she has published a life story book on rare diseases to raise awareness and advocate for those in need.

Picture of Lo Yat Hei

Lo Yat Hei

Dr. Lo Yat Hei is an emergency physician who is trained and grew up in Hong Kong. He now serves at the Accident and Emergency Department of Prince of Wales Hospital and teaches at the Accident and Emergency Medicine Academic Unit of the Chinese University of Hong Kong. When not practicing medicine, he enjoys gardening, ceramics and playing mahjong.

Listen to the chapter

References

  1. American Psychiatric Association. Diagnostic and Statistical Manual of Mental Disorders, Fifth Edition (DSM-5). American Psychiatric Association; 2013.
  2. World Health Organization. “Dementia.” Fact Sheet, https://www.who.int/news-room/fact-sheets/detail/dementia.
  3. Ljubenkov PA, Geschwind MD. Dementia. Semin Neurol. 2016;36(4):397-404. doi:10.1055/s-0036-1585096
  4. Plum F. The pathophysiology of dementia. Gerontology. 1986;32 Suppl 1:67-72. doi:10.1159/000212832
  5. Wilson JE, Mart MF, Cunningham C, et al. Delirium [published correction appears in Nat Rev Dis Primers. 2020 Dec 1;6(1):94]. Nat Rev Dis Primers. 2020;6(1):90. Published 2020 Nov 12. doi:10.1038/s41572-020-00223-4
  6. Maclullich AM, Ferguson KJ, Miller T, de Rooij SE, Cunningham C. Unravelling the pathophysiology of delirium: a focus on the role of aberrant stress responses. J Psychosom Res. 2008;65(3):229-238. doi:10.1016/j.jpsychores.2008.05.019
  7. Fong TG, Tulebaev SR, Inouye SK. Delirium in elderly adults: diagnosis, prevention and treatment. Nat Rev Neurol. 2009;5(4):210-220. doi:10.1038/nrneurol.2009.24
  8. Avelino-Silva TJ, Campora F, Curiati JAE, Jacob-Filho W. Prognostic effects of delirium motor subtypes in hospitalized older adults: A prospective cohort study. PloS one. 2018;13(1):e0191092. doi:10.1371/journal.pone.0191092
  9. Emmady PD, Schoo C, Tadi P. “Major Neurocognitive Disorder (Dementia).” In: StatPearls. StatPearls Publishing; 2023. Available from: https://www.ncbi.nlm.nih.gov/books/NBK557444/
  10. Morandi A, Davis D, Bellelli G, et al. The Diagnosis of Delirium Superimposed on Dementia: An Emerging Challenge.  J Am Med Dir Assoc. 2017;18(1):12–18. doi:10.1016/j.jamda.2016.07.014
  11. Han JH, Suyama J. Delirium and Dementia. Clin Geriatr Med. 2018;34(3):327-354. doi:10.1016/j.cger.2018.05.001
  12. Han JH, Wilson A, Ely EW. Delirium in the older emergency department patient: a quiet epidemic. Emerg Med Clin North Am. 2010;28(3):611-631. doi:10.1016/j.emc.2010.03.005
  13. Brodaty H, Connors MH. Pseudodementia, pseudo-pseudodementia, and pseudodepression. Alzheimers Dement. 2020;12(1):e12027. doi:10.1002/dad2.12027
  14. Ross GW, Bowen JD. The diagnosis and differential diagnosis of dementia. Med Clin North Am. 2002;86(3):455-476. doi:10.1016/s0025-7125(02)00009-3
  15. Carpenter CR, Banerjee J, Keyes D, et al. Accuracy of Dementia Screening Instruments in Emergency Medicine: A Diagnostic Meta-analysis. Acad Emerg Med. 2019;26(2):226-245. doi:10.1111/acem.13573
  16. Nasreddine ZS, Phillips NA, Bédirian V, et al. The Montreal Cognitive Assessment, MoCA: a brief screening tool for mild cognitive impairment. J Am Geriatr Soc. 2005;53(4):695–699. doi:10.1111/j.1532-5415.2005.53221.x
  17. Barr J, Fraser GL, Puntillo K, et al. Clinical practice guidelines for the management of pain, agitation, and delirium in adult patients in the intensive care unit. Critical Care Medicine. 2013;41(1):263-306. doi: 10.1097/CCM.0b013e3182783b72
  18. Hall JE, Uhrich TD, Barney JA, Arain SR, Ebert TJ. Sedative, amnestic, and analgesic properties of small-dose dexmedetomidine infusions. Anesth Analg. 2000;90(3):699-705. doi: 10.1097/00000539-200003000-00033
  19. Lightfoot CB, Breden C, Moczygemba LR. Delirium: diagnosis, prevention and management. Am J Health Syst Pharm. 2017;74(18):1365-1375. doi: 10.2146/ajhp160950
  20. Lempert T, Schmidt D, Rosemeyer J. Psychogenic nonepileptic seizures: a guide. J Neurol Neurosurg Psychiatry. 2006;77(2):297-303. doi:10.1136/jnnp.2005.082149.
  21. Rossor MN, Fox NC, Mummery CJ, Schott JM, Warren JD. The diagnosis of young-onset dementia. Lancet Neurol. 2010;9(8):793–806. doi:10.1016/S1474-4422(10)70159-9

Reviewed and Edited By

Picture of Joseph Ciano, DO, MPH, MS

Joseph Ciano, DO, MPH, MS

Dr. Ciano is a board-certified attending emergency medicine physician from New York, USA. He works in the Department of Emergency Medicine and Global Health at the Hospital of the University of Pennsylvania. Dr. Ciano’s global work focuses on capacity building and medical education and training in low-middle income countries. He is thrilled to collaborate with the iEM Education Project in creating free educational content for medical trainees and physicians.

Picture of Arif Alper Cevik, MD, FEMAT, FIFEM

Arif Alper Cevik, MD, FEMAT, FIFEM

Prof Cevik is an Emergency Medicine academician at United Arab Emirates University, interested in international emergency medicine, emergency medicine education, medical education, point of care ultrasound and trauma. He is the founder and director of the International Emergency Medicine Education Project – iem-student.org, chair of the International Federation for Emergency Medicine (IFEM) core curriculum and education committee and board member of the Asian Society for Emergency Medicine and Emirati Board of Emergency Medicine.

Question Of The Day #50

question of the day

Which of the following is the most appropriate next step in management for this patient’s condition?

This patient presents to the Emergency Department with altered mental status.  This presenting symptom can be due to a large variety of etiologies, including hypoglycemia, sepsis, toxic ingestions, electrolyte abnormalities, stroke, and more.  The management and evaluation of a patient with altered mental status depends on the primary assessment of the patient (“ABCs”, or Airway, Breathing, Circulation) to identify any acute life-threatening conditions that need to be managed emergently, the history, and the physical examination.  One mnemonic that may help in remembering the many causes of altered mental status is “AEIOUTIPS”.  The table below outlines this mnemonic.

ALTERED MENTAL STATUS

This patient arrives hyperthermic, tachycardic in atrial fibrillation, diaphoretic, and altered with psychotic behavior.  Thyroid storm, the most severe manifestation of hyperthyroidism, should always be on the differential diagnosis in patients with fever and altered mental status.  Other considerations are sepsis, sympathomimetic overdose, anticholinergic overdose, serotonin syndrome, and pheochromocytoma. 

This patient has thyroid storm, a life-threatening endocrine emergency that requires prompt recognition and treatment.  Symptoms of thyroid storm include altered mental status, psychosis, seizures, coma, tachycardia, atrial fibrillation, high-output heart failure, dyspnea, vomiting, diarrhea, weight loss, and anterior neck enlargement.  Severe hyperthyroidism should have a low-undetectable TSH level with elevated T3/T4 levels, but in acute illness these levels may be unreliable.  For this reason, the diagnosis and treatment of thyroid storm should be based on clinical grounds.

An anticholinergic toxidrome can appear similar to this patient with tachycardia, hypertension, agitation, and altered mental status.  A key differentiating factor is diaphoresis.  Patients with anticholinergic ingestions should have dry skin, not wet skin. The treatment for anticholinergic toxicity is benzodiazepines and IV physostigmine (Choice A) if symptoms are unresponsive to benzodiazepines.  Physostigmine is not the best next step in this scenario. 

Treatment of thyroid storm is algorithmic.  First, beta blockade (Choice C) should be given to control the heart rate and block T4 to T3 conversion, next anti-thyroid medications (Methimazole or Propylthiouracil (Choice D)) should be given to block thyroid hormone synthesis, and lastly corticosteroids and inorganic iodine (Choice B) can be given to block release of stored thyroid hormone.  The best next step in managing this patient with thyroid storm is administration of IV Propranolol (Choice C).  Propranolol helps manage the tachycardia, systemic symptoms, and also inhibits conversion of T4 to T3. 

 Correct Answer: C

References

[cite]

Question Of The Day #49

question of the day
Which of the following is the most likely cause of this patient’s condition?

This patient presents to the Emergency Department with altered mental status.  This presenting symptom can be due to a large variety of etiologies, including hypoglycemia, sepsis, toxic ingestions, electrolyte abnormalities, stroke, and more.  The management and evaluation of a patient with altered mental status depends on the primary assessment of the patient (“ABCs”, or Airway, Breathing, Circulation) to identify any acute life-threatening conditions that need to be managed emergently, the history, and the physical examination.  One mnemonic that may help in remembering the many causes of altered mental status is “AEIOUTIPS”.  The table below outlines this mnemonic.

ALTERED MENTAL STATUS

Hyperthermia (or hypothermia) can cause altered mental status.  This patient arrives with altered mental status, severe hyperthermia, tachycardia, tachypnea, and hypotension.  The history of the patient running outside for exercise should raise concern for hyperthermia related to excess heat production due to overexertion.  This should narrow the differential diagnoses to heat exhaustion (Choice B) and heat stroke (Choice C).  Both heat exhaustion and heat stroke are marked by hyperthermia with temperatures often over 40ᵒC. Additional symptoms include weakness, nausea, vomiting, myalgias, syncope, and headache.  The differentiating factor between heat exhaustion and heat stroke is altered mental status and sweating.  Patients with heat exhaustion lack altered mental status and should still be able to thermoregulate through sweating.  On the contrary, heat stroke patients are more severely ill as they have altered mental status and can no longer thermoregulate with sweating.  The treatment in both conditions should be early and aggressive cooling measures.  This includes full body immersion in an ice bath, removal of clothes, and cold IV fluids.  Internal cooling with gastric, bladder, pleural, or peritoneal lavage with cold fluids can be done on more sick patients.  Antipyretic medications, like NSAIDs and paracetamol, have no benefit in patients with severe hyperthermia.  Evaluation for rhabdomyolysis, kidney failure, liver failure, sepsis, or other organ dysfunction should also be a part of the evaluation of hyperthermic patients.

Sympathomimetic toxicity (Choice A) is possible, but less likely as the skin is dry and the history of exercise outdoors.  Sympathomimetic toxicity manifests as diaphoresis, tachycardia, hypertension, hyperthermia, and sometimes altered mental status.  Thyroid storm (Choice D) is another possibility.  This diagnosis can also present with similar vital signs, hyperthermia, and altered mental status.  Again, the history of outdoor exercise should point more towards heat exhaustion vs heat stroke.

The diagnosis of this patient is heat stroke (Choice C) as he has altered mental status and lacks wet skin.

Correct Answer: C

References

[cite]

Question Of The Day #48

question of the day

Which of the following is the most likely cause of this patient’s condition?

This patient presents to the Emergency Department with altered mental status.  This presenting symptom can be due to a large variety of etiologies, including hypoglycemia, sepsis, toxic ingestions, electrolyte abnormalities, stroke, and more.  The management and evaluation of a patient with altered mental status depends on the primary assessment of the patient (“ABCs”, or Airway, Breathing, Circulation) to identify any acute life-threatening conditions that need to be managed emergently, the history, and the physical examination.  One mnemonic that may help in remembering the many causes of altered mental status is “AEIOUTIPS”.  The table below outlines this mnemonic.

ALTERED MENTAL STATUS

The first step in managing this patient should be to treat the hypoxia with supplemental oxygen.  Prolonged hypoxia is dangerous and if left untreated, can cause brain damage.  Hypoxia can cause altered mental status, however, when this patient’s hypoxia is resolved, she remains somnolent and altered.  This should raise concern over an alternative etiology for the patient’s condition.      

The arterial blood gas demonstrates a low pH (acidosis), normal paO2, elevated paCO2 (hypercarbia), and a normal HCO3 (no metabolic compensation for acidosis).  The final interpretation of the ABG would be an acute respiratory acidosis without metabolic compensation.  Acute elevations of pCO2 can manifest as somnolence and altered mental status as seen in this patient.  This is known as hypercarbic or hypercapnic respiratory failure (Choice A).  This condition is caused by the inability to exhale CO2.  Risk factors include obstructive lung diseases (i.e., COPD), obesity, and obstructive sleep apnea.  Treatment involves treatment of hypoxia with supplemental oxygen, non-invasive positive pressure ventilation (i.e., BIPAP, CPAP, High Flow Nasal Cannula), and treatment of the underlying cause.

The patient’s arterial blood gas does not show hypoxic respiratory failure (Choice B).  Since treatment of the patient’s hypoxia does not improve the patient’s mental status, hypercarbic respiratory failure is more likely the underlying cause of the patient’s condition.  Opioid overdose (Choice C) can cause a similar ABG and patient presentation.  The normal size pupils and absent history of drug abuse makes this diagnosis less likely. Sepsis (Choice D) can trigger changes in mental status and cause respiratory failure, however, the absence of infectious symptoms and the presence of obesity and COPD support hypercarbic respiratory failure as the more likely underlying cause. 

Correct Answer: A

References

[cite]

Question Of The Day #47

question of the day

Which of the following is the most appropriate next step in management for this patient’s condition?

This patient presents to the Emergency Department with altered mental status.  This presenting symptom can be due to a large variety of etiologies, including hypoglycemia, sepsis, toxic ingestions, electrolyte abnormalities, stroke, and more.  The management and evaluation of a patient with altered mental status depends on the primary assessment of the patient (“ABCs”, or Airway, Breathing, Circulation) to identify any acute life-threatening conditions that need to be managed emergently, the history, and the physical examination.  One mnemonic that may help in remembering the many causes of altered mental status is “AEIOUTIPS”.  The table below outlines this mnemonic.

ALTERED MENTAL STATUS

The initial approach to all Emergency Department patients, especially those with abnormal vital signs, should include a primary survey (“ABCs”, or Airway, Breathing, Circulation).  This patient is breathing independently but at a significantly reduced rate and is hypoxic.  Hypoxia should prompt the administration of supplemental oxygen to the patient and reassessment of the SpO2.  The patient’s reduced respiratory rate, lethargy, and bilateral miosis (constricted pupils) should strongly hint at the possibility of opioid overdose.  Although the patient is lethargic and hypoxic, establishing a definitive airway (endotracheal intubation) should be avoided until after the antidote to opioid overdose is administered.  Naloxone is a mu-opioid receptor antagonist and functions as the antidote to opioid overdose.

 

Administration of 1000mL of 0.9% NaCl (Choice A) is unlikely to fix the patient’s clinical condition.  The patient needs naloxone to improve respiratory status.  25g of IV dextrose (Choice B) would be helpful if this patient’s altered mental status was from hypoglycemia.  A normal glucose level is provided in the question stem.  100mg of IV thiamine (Choice D) may be helpful in the case of Wernicke-Korsakoff Syndrome, a state of thiamine deficiency often associated with malnutrition and alcohol abuse.  Wernicke-Korsakoff Syndrome presents with vision disturbances, ataxia, and confusion.  Typically, this syndrome does not present with severe lethargy or depressed mental status as is seen in this patient.

The best next step in management is 1mg of IV naloxone (Choice C).  If given appropriately, naloxone can prevent the need for intubation.  Naloxone has a very short onset to action (~1min).  If suspicion for opioid overdose is high and there is an inadequate respiratory response after a single naloxone dose, repeat doses of naloxone are appropriate.  Naloxone can be administered in repeat boluses every 3-minutes to a total dose of 10mg IV.  Patients who respond appropriately to naloxone should be observed for recurrent respiratory depression as naloxone is cleared.  Need for repeat doses of naloxone indicates the need for a continuous naloxone infusion and hospital admission.  The typical infusion dose is 2/3 the “wake-up” dose given over 1 hour as a continuous infusion.  For example, if the patient responded to 1mg IV initially, the continuous infusion dose would be 0.6mg/hour of IV naloxone.

Correct Answer: C

References

[cite]

Question Of The Day #46

question of the day

Which of the following is the most appropriate next step in management for this patient’s condition?

This patient presents to the Emergency Department with altered mental status.  This presenting symptom can be due to a large variety of etiologies, including hypoglycemia, sepsis, toxic ingestions, electrolyte abnormalities, stroke, and more.  The management and evaluation of a patient with altered mental status depends on the primary assessment of the patient (“ABCs”, or Airway, Breathing, Circulation) to identify any acute life-threatening conditions that need to be managed emergently, the history, and the physical examination.  One mnemonic that may help in remembering the many causes of altered mental status is “AEIOUTIPS”.  The table below outlines this mnemonic.

ALTERED MENTAL STATUS

The serum chemistry results provided show elevated BUN and Creatinine with a BUN/Cr ratio of 21.3.  A BUN/Cr ratio greater than 20 indicates decreased perfusion to the kidneys, also known as pre-renal azotemia, which can indicate dehydration, hypovolemia, or shock.  The serum chemistry also shows a severely low sodium level.  Hyponatremia can present with a variety of symptoms, including weakness, fatigue, myalgias, nausea, vomiting, headaches, altered mental status, focal neurologic deficits, seizures, or coma.  Hyponatremia can be acute or chronic, asymptomatic or symptomatic, and mild or severe.  Sodium levels below 120 mEq/L are severely low.  Neurologic symptoms, such as seizures, altered mental status, and focal neurologic deficits, are also considered severe.  Treatment should be based on patient symptoms, rather than the sodium level, as it can be difficult to assess how acute or chronic the hyponatremia state is on initial evaluation.  The presence of any severe neurologic symptoms as is seen in this scenario should prompt administration of hypertonic saline (3% NaCl).  This allows for rapid correction of serum sodium levels, which should in turn relieve the neurologic symptoms.  A 100-150mL IV bolus of 3% NaCl can be given a second time if symptoms continue after 5-10 minutes.  

Typically, hyponatremia should be corrected slowly to avoid central pontine myelinolysis.  Increases in sodium greater than 8mEq/L per 24hours should be avoided for this reason.  However, in the case of neurologic symptoms, rapid correction of sodium is opted for to prevent further damage.

Administration of “normal saline”, or 1000mL of IV 0.9% NaCl (Choice A), can increase the sodium level.  However, normal saline is not concentrated enough to rapidly increase the serum sodium to terminate neurologic symptoms.  A noncontrast CT scan of the head (Choice B) is a reasonable investigation for this altered patient, but hypertonic saline should be administered first if hyponatremia is known.  Administration of 25mg IV dextrose (Choice C), also known as “D50”, would be helpful in a patient with hypoglycemia and altered mental status. However, this patient is not hypoglycemic.

Administration of hypertonic saline (Choice D) is the best next step in this patient with severe hyponatremia and neurologic symptoms.

Correct Answer: D

References

[cite]

Question Of The Day #45

question of the day

Which of the following is the most appropriate next step in management for this patient’s condition?

This patient presents to the Emergency Department with altered mental status.  This presenting symptom can be due to a large variety of etiologies, including hypoglycemia, sepsis, toxic ingestions, electrolyte abnormalities, stroke, and more.  The management and evaluation of a patient with altered mental status depends on the primary assessment of the patient (“ABCs”, or Airway, Breathing, Circulation) to identify any acute life-threatening conditions that need to be managed emergently, the history, and the physical examination.  One mnemonic that may help in remembering the many causes of altered mental status is “AEIOUTIPS”.  The table below outlines this mnemonic.

ALTERED MENTAL STATUS

This patient’s altered mental status is likely due to a post-ictal state after a first-time seizure.  A seizure occurs when the brain is in a state of neuronal hyperactivity.  First time seizures can be caused by a variety of factors, such as hypoxia, hyperthermia, hypoglycemia, traumatic brain injury, brain tumors, meningitis, encephalitis, hyponatremia, or alcohol withdrawal.  It can sometimes be difficult to differentiate a seizure from a syncopal episode.  Both conditions cause loss of consciousness and both may include body convulsions.  Details that support a diagnosis of seizure over syncope include bowel or bowel incontinence, tongue biting, and confusion after regaining consciousness (post-ictal state).

Management of a patient having a seizure should focus initially on the ABCs (Airway-Breathing-Circulation) and terminating the seizure.  This involves first repositioning the patient to prevent aspiration.  A common maneuver is rolling the patient in the lateral decubitus position, performing a jaw thrust, and suctioning the airway (Choice C).  Administration of IM haloperidol (Choice A) is unlikely to terminate the seizure as it is an antipsychotic, not an antiepileptic medication.  Obtaining a 12-lead EKG (Choice D) is an important aspect of evaluating a patient with a potential seizure, however, the next best step in this seizing patient should focus on the ABCs and terminating the seizure.  Endotracheal intubation (Choice B) may be necessary in this patient to protect the airway, but patient repositioning (Choice C) and antiepileptic (i.e., benzodiazepines) administration are important initial steps prior to considering intubation.  The best next step in this scenario is Choice C.

 Correct Answer: C

References

[cite]

Question Of The Day #44

question of the day

Which of the following is the most appropriate next investigation to confirm this patient’s diagnosis?

This patient presents to the Emergency Department with altered mental status.  This presenting symptom can be due to a large variety of etiologies, including hypoglycemia, sepsis, toxic ingestions, electrolyte abnormalities, stroke, and more.  The management and evaluation of a patient with altered mental status depends on the primary assessment of the patient (“ABCs”, or Airway, Breathing, Circulation) to identify any acute life-threatening conditions that need to be managed emergently, the history, and the physical examination.  One mnemonic that may help in remembering the many causes of altered mental status is “AEIOUTIPS”.  The table below outlines this mnemonic.

ALTERED MENTAL STATUS

The information provided indicates that the patient’s headache was maximal at onset, severe, associated with vomiting, and led to a deteriorating mental status ultimately requiring intubation.  This history is very concerning for intracranial bleeding, especially subarachnoid hemorrhage (SAH).  The majority of atraumatic SAHs are caused by the rupture of a saccular aneurysm.  This causes the leakage of blood into the subarachnoid space.  Symptoms of a SAH are sudden onset headache that is maximal intensity at onset (“thunderclap headache”), syncope, vomiting, seizures, and any neurological deficits.  Risk factors for SAH are age over 50years-old, family history of SAH, alcohol abuse, tobacco smoking, Marfan Syndrome, Ehlers-Danlos Syndrome, and Polycystic Kidney Disease.  Diagnosis of SAH takes into account the patient’s history, physical exam, and risk factors. 

Patients that arrive in the Emergency Department under 6hours since symptom onset should initially get a noncontrast CT scan of the head (Choice D).  When a noncontrast head CT is performed in this time window, its sensitivity reaches 98-100%.  Noncontrast head CTs performed within the first 24hrs since headache onset have a sensitivity of about 90%.  Patients with signs and symptoms concerning for SAH who have a negative CT head should get a lumbar puncture (Choice A) to evaluate for xanthochromia.  This is especially important if the patient’s symptoms have been for over 6 hours.  A 12-lead EKG (Choice B) can show ST and T wave changes, but an EKG alone cannot be used to make a diagnosis of SAH.  A brain MRI (Choice C) can make the diagnosis of SAH, but a CT scan would be preferred due to greater CT scan accessibility, cost, and the shorter time of this imaging test.  The best next investigation would be a noncontrast CT of the head (Choice D).

Correct Answer: D

References

[cite]

Question Of The Day #43

question of the day

Which of the following is the most likely cause for this patient’s altered mental status?

This patient presents to the Emergency Department with altered mental status and fever.  Altered mental status can be due to a large variety of etiologies, including hypoglycemia, sepsis, toxic ingestions, electrolyte abnormalities, stroke, and more.  The management and evaluation of a patient with altered mental status depends on the primary assessment of the patient (“ABCs”, or Airway, Breathing, Circulation) to identify any acute life-threatening conditions that need to be managed emergently, the history, and the physical examination.  One mnemonic that may help in remembering the many causes of altered mental status is “AEIOUTIPS”.  The table below outlines this mnemonic.

ALTERED MENTAL STATUS

This patient has confusion, fever, lower abdominal pain, dysuria, and no focal neurological deficits on exam.  Diabetic ketoacidosis (Choice A) is unlikely as the patient does not have marked hyperglycemia (>250mg/dL (13.8mmol/L)), polyuria, or polydipsia.  Intracranial hemorrhage (Choice C) is unlikely as the patient has no headache, history of trauma, focal neurologic deficits, or coma.  Severe hypothyroidism (Choice D), known as myxedema coma, can cause altered mental status.  This condition is marked by somnolence or coma, hypothermia, nonpitting edema on the hands and feet, dry skin, macroglossia (enlarged tongue), and hair loss.  This patient does not have symptoms consistent with severe hypothyroidism. 

Sepsis (Choice B), especially in elderly individuals, can cause altered mental status.  The patient’s fever, confusion, lower abdominal pain, and dysuria all point to a likely diagnosis of urosepsis.  Sepsis is the most likely cause of this patient’s disoriented state.  Treatment with early IV hydration and antibiotics will help remedy the patient’s altered mental status.  Correct Answer: B

References

[cite]